Набор прочности бетона во времени снип: График набора прочности бетона (СНИП)

Содержание

График набора прочности бетона (СНИП)

Содержание

  1. Этапы твердения раствора
  2. Что влияет на набор максимальной прочности
  3. Ускорение набора прочности

Ключевой этап проведения ремонтно-строительных работ – сушка бетона. Залитый состав отвердевает и набирает прочность несколько недель. Процесс проходит под наблюдением инженеров и требует постоянного контроля.

Специалисты обеспечивают выполнение нормативов и при необходимости вносят коррективы в график. Материал чувствителен к температурным колебаниям и имеет «коэффициент сезонности» – зимой бетонные работы проводят с использованием систем обогрева. Чтобы определить, сколько сохнет бетон, учитывают различные факторы.

Этапы твердения раствора

Бетонные работы – часть любого строительства, от дачно-коттеджного до промышленного и специального. Материал применяют на различных стадиях возведения объектов, для заливки фундамента и несущих конструкций, устройства перекрытий.

Строители успешно используют свойство цементно-песчаной смеси с добавлением щебня – способность принимать форму опалубки. Ценят прочность и долговечность материала, время высыхания которого составляет порядка 28 дней.

В зависимости от условий эксплуатации и качества состава расчетный срок службы объектов достигает 250 лет, а в среднем оценивается в 50-100. Для современного строительства это солидный период – технологии постоянно совершенствуются, появляются новые материалы и конструктивные решения.

Набору прочности по-прежнему уделяют особое внимание и контролируют каждый этап:

  1. Застывание. Происходит в первые часы «жизни» состава. К месту работ раствор доставляют в бетономешалке или подготавливают на месте для максимального сохранения необходимых свойств. Время застывания летом при температуре выше 20°С – около часа, в жару – 15-30 минут. При «ноле» – начинается через 6-10 часов после приготовления смеси и растягивается до 20 часов с момента заливки;
  2. Твердение. Основной этап занимает 7-14 дней. За этот период конструкция набирает до 70% расчетного значения, которое зависит от марки бетона;
  3. Контрольное значение по ГОСТ 18105-86. Стандартное время набора прочности – 28 дней. Специалисты сравнивают полученный результат с нормативами специальной таблицы.

Имеется прямая зависимость между затвердением раствора в различных условиях и достижением максимального значения.

Что влияет на набор максимальной прочности

Абсолютное большинство бетонных работ выполняют на открытом воздухе. Погодные условия и температурный график – ключевые параметры, которые определяют, сколько застывает раствор.

В теплое время года созревание смеси и постепенное отвердение происходит естественным образом. Процесс зависит от физико-химических свойств состава и имеет небольшие отличия, связанные с маркой бетона.

В осенне-зимний период набор прочности обеспечивают двумя способами:

  • Противоморозные добавки. Используют для сохранения свойств приготовленного раствора. Специальные вещества не допускают замерзание воды и потерю качества, облегчают заливку конструкции, выравнивание поверхности;
  • Электропрогрев. Выполняется несколькими методами с общей сутью – обеспечение равномерного прогрева толщи бетона в течение периода, необходимого для набора прочности.

При низких температурах применяют провода ПНСВ или «вживляют» в материал электроды, после чего подключают напряжение. Реже используют в качестве нагревательного элемента саму опалубку, покрывают поверхность специальными матами.

Работы требуют соблюдения правил электробезопасности и выполняются по СНиП 3.03.01-87. Если минимальная температура достигает 0°С, а средняя за сутки не превышает 5°С, бетонирование изначально планируют с прогревом залитой конструкции. При необходимости в раствор включают ПМД.

Ускорение набора прочности

Бетонные составы классифицируют в зависимости от показателя прочности на сжатие. Легкие растворы используют для вспомогательных работ или конструкций, которые не испытывают нагрузку.

Базовыми считаются бетоны М-200 – М-400. Составы применяют при сооружении большинства объектов гражданского строительства. Растворы класса выше М-500 предназначаются для специальных объектов и конструкций повышенной прочности.

Базовую скорость отвердения рассчитывают на основе марок М-200 – М-300. Показатели основаны на временном промежутке в четыре недели. На практике необходимый период сокращается при определенных условиях:

  • Использование специальных добавок. Это вспомогательные компоненты, которые подмешивают в раствор при приготовлении. Применение сокращает время полного застывания до 14 дней. Такие работы проводят летом – антиморозные добавки не обладают подобным свойством;
  • Увлажнение. При сухой жаркой погоде происходит быстрое испарение воды из высыхающего состава, что отрицательно влияет на график набора прочности и качество конструкции. Постоянное увлажнение способствует созданию условий, при которых достигают оптимальной динамики застывания.

После завершения расчетного периода проводят испытания бетона и контрольные замеры. Если показатели соответствуют нормативам, приступают к следующим этапам работ.

Чтобы строительство завершилось согласно планам, рекомендуется разработать детальную проектную документацию с учетом особенностей конструкции. В календарном графике бетонные работы по возможности планируют в наиболее благоприятный сезон.

Набор прочности бетона: особенности ускорителей, по СНиП




Наиважнейшим свойством для бетонов, характеризующим его марку, является прочность изделия, которая проверяется в лабораторных условиях. Образец, достигший возраста 28 суток (срок, за который осуществляется набор прочности бетона в зависимости от температуры,может изменяться), подвергается нагрузке на сжатие до первых признаков разрушения. По результатам испытаний назначается марка бетона в условиях сжатия в диапазоне от М50 до М800.

Не все задумываются о том, а будет ли изделие обладать необходимой прочностью, слепо доверяя наемным рабочим или инструкции рабочего процесса, которую подсказал сосед

Проектирование объектов строительства

При проектировании строительства с применением бетонных смесей утверждается определенный класс материала для каждой отдельно взятой конструкции.

Использование той или иной марки зависит от области применения, но можно смело утверждать, что самыми ходовыми в градостроительстве считаются марки в диапазоне 100-500.

Обычно именно кубик является результатом лабораторных исследований

  • Проектируя сооружения, где недопустимо образование трещин в опорных элементах (плотин, резервуарах запаса воды и т.п.) следует учитывать прочность затвердевшего раствора на растяжение.
  • Проектирование дорожных покрытий и взлетных линий аэродромов большое значение имеет показатель прочности на растяжение при изгибе.

Все эти составляющие показателя прочности бетонов отражаются в маркировке материала.

Для понимания приведем соотношение показателей марочности и классности бетона с показателями прочности смеси классов:

  • В 3,5 – средний показатель прочности 46 кгс/см2(ниже будут указаны только числовые значения)- марка 50.
  • В 5 — 65 — М 75.
  • В 7,5 – 98 — М 100.
  • В 10 и В 12,5 — 131 и 164 — М 150.
  • В 15 – 196 — М 200.
  • В 20 — 262 — М 250.
  • В 25 – 327 — М 300.
  • В 30 — 393 — М 450.
  • В 35 и В 40 — 458 и 524 — М 550.
  • В 45 и В 50 – 589 и 655- М 600.
  • В 55 – 720 — М 700.
  • В 60 – 786 — М 800.

Перевести значение марки смеси в класс можно по формуле — В=(М*0,787)/10. Если вы собираетесь своими руками что-либо возводить, то есть является частным застройщиком, то оптимально использовать марки 250 – 550.

Наличие множества трещин и пузырьков говорит о низком качестве изделия

К сведению!
Класс бетона отражает гарантированную прочность в Мпа.
Изменять класс бетона на этапе строительства в разрез с проектом возможно только по согласованию с проектной организацией и утверждения в отделе архитектурного надзора.
К этому прибегают в исключительных случаях.

Что влияет на прочность бетона

На данный показатель готового изделия влияют многие факторы:

Качественный и правильный армокаркас (на фото) – еще один показатель, от которого напрямую зависит прочность бетонного изделия

  • Активность и содержание цемента в смеси.
  • Водоцементное отношение.
  • Выбор минеральных заполнителей.
  • Степень однородности (зависит от качества перемешивания).
  • Последующее уплотнение.
  • Время, прошедшее после заливки.
  • Условия отвердевания бетона.

Для сокращения сроков твердения бетонных изделий используют ускорители набора прочности бетона. Их применяют по технологической необходимости и всегда при производстве работ в зимнее время. С введением в состав смеси специальных добавок, конечно же, повышается отпускная стоимость бетона.

Важно!
Добавки актуальны только для крупных строительных компаний, занимающихся возведением многоквартирных жилых домов, частным застройщикам особой пользы они не принесут.

Достижение бетоном марочной прочности

Показания степени готовности смеси в зависимости от временных интервалов отражает график набора прочности бетона СНиП. В нормативе оговариваются средние сроки твердения и время набора прочности бетона.

При нормальных условиях смесь созревает в течение 28 суток, причем:

По горизонтали указано время (сутки), по вертикали прочность (проценты)

  • Первый этап интенсивного твердения – первые пять дней.
  • По истечении семи дней после завершения заливки достигается 70% проектной прочности изделия.
  • Окончательного срока отвердевания (100%) придется ожидать оставшиеся 21 сутки и только тогда можно приступать к дальнейшему ведению работ.

Внимание!
Так изменяются показатели прочности бетонных растворов во времени по СНиПу.
В реальных условиях они могут меняться в связи с разными причинами, на это влияет не только температура, но и качество раствора.

Уход в период выдержки

Вот небольшая инструкция о методах создания оптимальных условий для созревания цемента:

  • Первые пять- семь дней особенно важное проведение мероприятий по обеспечению комфортных условий выдержки залитого цемента:
    • Поверхность раствора накройте влагозащитным материалом (подойдет обычная полиэтиленовая пленка, ее цена не сильно бьет по бюджету).
    • Активно увлажняйте ее примерно раз в сутки, вновь накрывая защитным полотном.
    • При необходимости организуйте прогрев бетона с помощью тепловых пушек.
  • Через неделю особое внимание уделяйте увлажнению, если на первоначальной стадии допускалось поливать твердеющий раствор раз за двое суток, то здесь пропускать нельзя.

Примечание!
При наружной температуре воздуха 25-30°С изделие можно вводить в эксплуатацию спустя 11 дней.

В нормативных документах, касающихся бетонных работ можно найти график набора прочности бетона В25 или любого другого класса, что поможет вам разобраться со сроками строительства.

Вот такие цифры и расчеты вы можете обнаружить в специализированных документах

Вывод

Надеемся, что вышеизложенная информация была вам полезна, напомним лишь, что набор прочности бетоном по времени СНиП гарантирован в идеальных условиях. Руководствуясь этими сведениями, следует давать поправку на «жизнь», и не ограничивать срок твердения при умеренной погоде 14-15 днями, как это может быть указано в документах.

В представленных видео в этой статье вы найдете дополнительную информацию по данной теме.


Время набора прочности бетона через 7 суток, график набора требуемой прочности СНИП в Санкт Петербурге


Качество бетона оценивается по показателю прочности. От него будет зависеть, сколько прослужит построенное здание, сооружение или будущая конструкция. Изготавливаться бетон должен обязательно по установленным требованиям ГОСТа и должны соблюдаться условия сжатия, обычно они варьируются в диапазоне М50-800. Самым востребованным считается цемент марки М100-500. При производстве бетона каждая партия тщательно исследуется и проверяется на все характеристики. Но в основном потребители считают нужным испытывать материал на прочность. Во время испытаний обязательно нужно выдерживать время набора прочности бетона. Только так можно получить точные данные.

Классификация бетона

Материал выпускается разного вида. Но есть три основных:

  1. Тяжелые составы. Их делают из традиционных плотных заполнителей и цементов (М50-800).
  2. Легкие составы. Используются пористые заполнители. Это бетоны М50-450.
  3. Ячеистые составы. Относятся к разряду легких и особо легких смесей (М50-150).

Устанавливается проектная марка бетона уже на этапах проектирования, когда ведется расчет и выбор всех конструкций. Требуемая прочность бетона получается на основании сопротивления осевому сжатию в контрольных образцах-кубиках. Для будущей конструкции наиболее важным является осевое растяжение и поэтому марку цемента определяют согласно сопротивлению на это растяжение.

Набор прочности бетона на растяжение растет с увеличением марки по прочности на сжатие (это указано в СНИПе), но надо учитывать, что в диапазоне высокопрочных материалов возможно замедление роста сопротивления.

Марка бетона и его класс прочности, а также состав определяется в зависимости от области применения. К менее прочным относят материалы, которые обозначены М50, М75, М100. Их используют тогда, когда конструкции не ответственны и им не требуется выдерживать сильные нагрузки.

Если при возведении зданий, сооружений важна большая прочность, то нужно использовать бетон марки М300.  Для стяжки неплохим вариантом является состав М200. Цементы от марок М500 относятся к самым крепким.

Важность графика набора прочности

Исследовательская работа по определению характеристик материала проводится согласно графику набора прочности бетона. Все эксплуатационные свойства раствор приобретает через определенное время, которое называется периодом выдерживания бетона. В графике отражается время, требуемое для достижения самого высокого значения прочности.

С того момента как заливается смесь по формам показатели прочности постоянно меняются. Так через 7 суток прочность бетона будет одной, а через 28 дней уже другой, более высокой. Срок 28 дней – это как раз то время, когда материал созревает полностью, но важно, чтобы для этого были созданы требуемые условия. Интенсивное твердение бетона происходит в первые 5 дней, а через 7 дней прочность достигается 70 процентов, но при этом нужно учитывать марку и класс. Специалисты рекомендуют дальнейшие строительные работы начинать только тогда, когда материал достигнет 100-процентной прочности и не раньше, то есть через 28 дней после заливки.

Есть такое понятие в строительстве, как распалубочная прочность бетона (его можно найти в СНИПе). То есть это самый минимальный показатель прочности, но при ней уже можно убрать опалубку и при этом материал не будет поврежден. Такая прочность вполне достаточна для транспортировки, но проводить строительные работы дальше пока нельзя. Надо выдержать конструкции еще минимум 28 дней.

Лаборанты отмечают, что время набора прочности может быть различным. Чтобы определить их точно, необходимо провести испытание образцов из каждой партии.

Если монолитное строительство происходит в теплое время года, то состав выдерживать для обретения оптимальных физических и механических свойств надо следующим образом:

  • Выдержать в опалубке бетона;
  • Дать дозреть составу после удаления опалубки.

Для холодного времени немного другие правила. Чтобы бетон приобрел требуемую прочность, то сам материал и его гидроизоляцию необходимо дополнительно обогревать. Потому что при низкой температуре процесс полимеризации проходит медленней.

Более подробно об этой информации можно узнать у специалистов в нашей компании (мы находимся в Санкт-Петербурге). У нас имеется специализированная лаборатория, и работают квалифицированные специалисты.

Процесс набора прочности бетона в зависимости от температуры (СНиП, ГОСТ)

Главное свойство бетонной смеси определяет набор прочности бетона, отражающий качественное состояние монолитной конструкции. Поскольку она находится во взаимосвязи со структурой данного строительного материала, то набор прочности можно поделить на два шага, связанных со схватыванием и затвердеванием бетона. Для последнего характерно наличие физико-химических свойств, возникающих при взаимодействии цемента с водой. Кода идет формирование бетона, то гидратация цемента вызывает образование других соединений.

Схема приготовления бетона.

Как происходит набор прочности бетона

Схватывание состава может произойти в первые дни с того момента, как была изготовлена консистенция из цемента и воды. Время ее схватывания находится в прямой зависимости от температуры воздуха. Если она составляет 20°С, то может понадобиться около одного часа. Поскольку процесс застывания бетона не мгновенный, а достаточно долговременный, то для набора прочности материала может потребоваться несколько месяцев.

Зачастую схватывание цемента происходит приблизительно спустя около двух часов с того момента, как был затворен цементный раствор, а окончательный процесс может начаться приблизительно спустя три часа. Поэтому на данной стадии может помочь ускоритель схватывания бетона.

Изображение 1. График набора прочности бетона.

Начало данной стадии может быть отодвинуто в результате снижения температурного уровня, а ее продолжительность существенно возрастает. Если уровень температуры воздуха составляет 0°С, то начало этапа схватывания может произойти спустя от 6 до 10 часов после того, как произошло затворение смеси. При этом данный процесс способен растянуться на 15-20 часов. Если температуры завышены, то период схватывания бетона может быть сокращен, что составит около 10-20 мин.

Схватывание бетона предполагает то, что данный состав должен оставаться подвижным весь период, что позволяет оказывать влияние на смесь. Механизм тиксотропии, связанный с уменьшением вязкости субстанции в условиях механического воздействия на нее, то есть периодического смешивания бетона, который схватился не полностью, твердение и процесс высыхания бетона не начинаются. Данное свойство учитывают в процессе доставки раствора на бетоносмесителе, поскольку состав при этом должен перемешиваться в миксере, что позволяет сохранять все его важные свойства.

Вращение миксера машины препятствует высыханию цементного раствора, не позволяя твердеть смеси достаточно долго. Возможно и развитие необратимых последствий, которые называют «свариванием» бетона, а это снижает его полезные свойства. Данный процесс особенно быстро может происходить летом.

Вернуться к оглавлению

Что представляет собой процесс твердения бетона

Ниже перечислены особенности, характерные для бетона:

Относительная прочность бетона в разные сроки твердения при различных температурах.

  1. Чем ниже уровень температуры внешней среды, тем медленней твердеет состав и нарастает его прочность.
  2. Если температура не превышает нулевую отметку по Цельсию, то вода в составе начинает замерзать, а твердение смеси уже не происходит. Повышение уровня температуры влечет за собой возобновление твердения.
  3. Влажность среды позволяет всей строительной массе приобретать более высокую прочность, чем в процессе затвердевания бетона вне помещения.
  4. Процесс схватывания бетона может стать замедленным и практически непрерывным при отсутствии влаги, так как именно она необходима в первую очередь при гидратации цемента.
  5. Если температура повышается до 80-90°С, то происходит значительное увеличение скорости процесса нарастания прочности в условиях максимальной влажности.

Пар высокого давления позволяет пропаривать смесь автоклавным способом, что осуществляется только при создании соответствующих условий.

Набор прочности бетона — это непостоянная величина. Если твердение бетона происходит в нормальных условиях, то набор прочности начинается через одну-две недели, что составляет от 60 до 70% от того уровня прочности, который набирается за 28 дней. Далее он продолжается, но очень медленно. С момента, когда была произведена заливка раствора, затвердевание бетона является максимальным.

При правильном течении процесса гидратации должны соблюдаться определенные условия. Уровень влажности должен составлять от 90 до 100%, а температуры — от 18 до 20°С. При нарушении данных условий может произойти изменение времени застывания состава.

Переход воды при отрицательных температурах в твердое состояние вызывает в результате промерзания бетона давление кристаллов льда на массу частиц цемента, что может снижать качество состава.

Таблица соответствия марок и классов бетона.

Смесь начинает затвердевать и при низком уровне влажности. Это вызвано прекращением поступления влаги, что требуется для гидратации цемента.

Если для конструкции характерны идеальные условия, то гидратация возобновляется. Когда подходит к концу уже вторая неделя, то смесь уже имеет прочность, составляющую 80% от основной первоначальной прочности. После этого ее набор замедляется.

На практике по истечении 28 дней завершение набора прочности не происходит, поскольку длительность данного процесса может составлять несколько лет. Когда смесь достигает трехлетнего возраста, то его прочность соответствует 200-250% от величины, характерной для возраста бетона, равного 28 суткам.

Никто не может дать однозначного ответа на вопрос о длительности процессов твердения смеси. Все зависит от той нагрузки, которая запланирована для той или иной конструкции.

Вернуться к оглавлению

Как осуществляют испытания

Например, если планируется строительство забора из металлического сайдинга либо досок, то для его возведения будет достаточно устройства бетонного ленточного фундамента. Если требуется начать строительство дома на бетонном фундаменте, то без помощи специалиста высокой квалификации здесь не обойтись. Процесс набора прочности в зависимости от температуры показан на рисунке (ИЗОБРАЖЕНИЕ 1).

Изображение 2. Таблица набора прочности бетона.

Марочная прочность, которая набрана за 28 суток, на рисунке взята за 100%. Оценка класса бетона производится спустя 28 суток. Осуществление процесса испытаний возможно с использованием образцов, имеющих стандартную кубическую форму. Сторона куба при этом может составлять 15 см. Температура, позволяющая выдержать образец, должна достигать 20°С, а относительная влажность колебаться в пределах 95%. Хранить смесь в виде испытуемых образцов можно в камере нормального хранения в нормальных условиях.

Если уровень температуры твердения отклоняется от нормального в наибольшую сторону, то созревание бетона будет осуществляться в условиях повышенной температуры. Если происходит ее отклонение к наименьшей стороне, то твердение бетона может предполагать сниженную температуру.

В таблице (ИЗОБРАЖЕНИЕ 2) отражена информация, связанная с набором прочности бетонного состава, имеющего марку от М200 до М300, изготавливаемого на основе портландцемента, маркой М-400 или М-500, за первые прошедшие 28 суток, что определяется среднесуточной температурой.

Вернуться к оглавлению

Способы заливки бетона при повышенных температурах

Среди многих факторов, оказывающих влияние на набор прочности бетонного раствора, в большей степени можно отметить следующие:

  1. Соотношение воды с цементом.
  2. Уровень уплотнения смеси.
  3. Тип цемента, необходимый при производстве раствора.
  4. Определенная температура, которая характерна в процессе твердения бетона.

Таблица критической прочности для разных марок бетона.

В подавляющем большинстве случаев, связанных с осуществлением работ с использованием раствора бетона, влияние атмосферных условий может быть слишком далеким от идеальных, поэтому необходимо принятие дополнительных мер. Когда заливка раствора осуществляется в холодный период, то отрицательные температуры требуют обеспечения прогрева смеси.

С этой целью можно применять ряд различных способов. Среди них можно выделить процесс прогрева бетона с применением электрических проводов. При этом заливку раствора делают, используя теплую опалубку. Для предотвращения процесса кристаллизации воды зимой в бетон производится ввод соответствующих антиморозных присадок.

В зимних условиях иногда может быть использован способ, который предполагает гидратацию цемента. С этой целью в бетон добавляют противоморозные вещества в небольших количествах. Температура при заливке смеси должна составлять не менее -15°С. Данные условия связаны с быстрым замерзанием воды и прекращением процесса гидратации, возобновление которого происходит только в весенний период. Применение данного метода способно приводить к процессу снижения качества бетонной конструкции.

Другое экстремальное условие связано с повышенным уровнем температуры окружающего воздуха. Данный случай позволяет увлажнять застывающий раствор. При этом после поливания раствора водой бетон должен быть укрыт специальной пленкой и слоем состава, который имеет битумную основу. Созревание бетона требует осуществления контроля над изменением объема смеси. Превышение в процентах не должно составлять 1% от первоначального уровня показателя.

Отсутствие усадки при этом является идеальным моментом, хотя на практике это не всегда становится возможным. При изменении объемов, которое имеет практическое значение, возможно применение специальных мер, далеко не всегда являющихся эффективными. Если времени на процесс высыхания бетона недостаточно, то на заливке могут появиться трещины, которые способны вызвать понижение прочности всей строительной конструкции.

График набора прочности бетона — таблица по суткам

Ключевым достоинством бетонных конструкций являются их высокие прочностные свойства и надежность. В зависимости от марки материал может использоваться в различных условиях. При этом степень набора прочности зависит от разных факторов.

Процесс набора

Бетон представляет собой популярный каменный материал, который создается на основе смеси воды, вяжущей добавки и заполнителя. В его состав вносятся специализированные добавки, отвечающие за особые свойства и функции.

В процессе гидратации происходит образование надежных монолитных соединений, которые приобретают свойства прочного искусственного камня. Для формирования монолита требуется несколько недель (до 28 суток), а получение заводских качеств занимает до 6 месяцев.

Созревание бетона состоит из 2 этапов:

  1. Схватывание. Является начальной стадией.
  2. Твердение. Финишная стадия.

Зная все нормы созревания, можно определить, сколько лет прослужит монолитная конструкция.

Схватывание

Использовать стройматериал сразу после заливки нельзя. Перед этим необходимо ознакомиться с графиком набора прочности бетона и спецификой каждого этапа его созревания. Нередко смесь доставляется на строительную площадку с помощью специальной техники, поэтому ее поддерживают в подвижном состоянии с помощью автоматизированного оборудования. Технология тиксотропии сохраняет базовые параметры консистенции до момента заливки, приостанавливая естественное созревание.

Но если выдержать смесь дольше допустимого времени или подвергнуть ее воздействию высоких температур, требуемые рабочие свойства будут ухудшены. В таблице набора прочности бетона упоминается, что он схватывается за период от 20 минут до 20 часов. Если работа выполняется при отрицательных температурах в зимнее время, термин увеличится до 6-10 часов.

Для защиты конструкции от деформации необходимо позаботиться о наличии теплой опалубки. Армированные элементы тщательно прогреваются и очищаются от льда. В летний период теплая опалубка малоэффективна.

Еще некоторые эксперты используют для зимних работ специализированные добавки и теплоизолирующие материалы. Выбирая этот вариант, необходимо ознакомиться с их свойствами и инструкцией по применению.

Для нагревания смеси можно использовать такие приспособления:

  1. Пар.
  2. Электроток.
  3. Известь-кипелку.
  4. Экзотермические цементы.
  5. Всевозможные ускорители.

Специалисты рекомендуют приступать к заливке раствора в формы при +20°C. В таком случае схватывание наступит через 1 час и займет не больше 60 минут. В жаркую погоду процесс происходит практически моментально.

Если применяются марки М300 и М200, а окружающая температура держится на отметке +20 °C, схватывающий процесс будет длиться в течение 1 часа.

Зная, сколько бетон набирает прочность, можно грамотно рассчитать время реализации проекта и определить приблизительные финансовые расходы.

Твердение

Следующий этап заключается в затвердевании бетонной смеси под воздействием гидратации. Процесс заключается в формировании из минералов цемента новых соединений. Если в составе раствора отсутствует влага, затвердевание будет замедлено или вовсе приостановлено, из-за чего материал не получит требуемую прочность и начнет растрескиваться.

При нормальном температурном режиме и достаточном количестве жидкости прочность будет постоянно расти. К благоприятным условиям относят температуру +20 °C и показатель влажности воздуха не меньше 90%.

Если такие требования соблюдены, процесс наращивания прочности составит 7-14 суток. За этот термин раствор получает 60-70% заявленной прочности, после чего процесс замедляется.

При выдерживании бетона в воде его прочностные свойства будут более высокими, чем при твердении на воздухе. Сухая среда способствует быстрому испарению влаги и остановке процесса. Это связано с тем, что зерна цементной смеси не успевают вступить в гидратацию. Поэтому, чтобы избежать неприятных последствий, необходимо исключить преждевременное высыхание бетона.

В процессе твердения монолита его объем постоянно меняется. Еще материал дает усадку — в поверхностных зонах она более быстрая, чем во внутренней части. В случае нехватки влажности при твердении на поверхности бетона появятся усадочные трещины. Дефекты возникают также при обильном тепловыделении.

Время набора прочности бетона зависит и от окружающей температуры. При низких отметках процесс замедляется, а при высоких — ускоряется.

Если возводимая конструкция будет подвергаться дополнительным нагрузкам или есть необходимость быстрее демонтировать опалубку, процесс твердения придется ускорить. Для таких задач задействуют специализированные добавки. Их концентрация определяется опытным путем в строительной лаборатории.

Чтобы получить заводскую прочность в сжатые сроки, необходимо правильно обслуживать раствор и поддерживать его во влажном состоянии, защищая от сотрясений, ударов и повреждений. При ненадлежащем уходе материал станет низкокачественным и уязвимым к растрескиванию.

Ключевой причиной нехватки прочности является низкая температура, которая сопровождает строителей при зимнем бетонировании.

Под воздействием холода возникают 2 проблемы:

  1. Замедление гидратации и рост сроков набора.
  2. Вымерзание жидкости из состава бетонной смеси, из-за чего набор прочностных свойств приостанавливается.

При низкой температуре сроки получения прочностных свойств сильно увеличиваются, поэтому к исходному сырью добавляют специальные компоненты.

В зимних условиях инженеры задействуют противоморозные добавки, которые запускают процессы набора и снижают температуру замерзания жидкого вещества.

При необходимости ускорить твердение при высокой температуре или повышенной влажности исходное сырье подвергается прогреву. После заливки смеси поверхность бетона нужно усилить матами или щитами, которые будут удерживать температуру от гидратации и сохранять требуемые условия. Если наполнитель замерзнет, его запрещено использовать для дальнейших работ.

Электрический прогрев бетона востребован на тех строительных площадках, где имеется доступ к трансформаторам с большой мощностью. Выполнение бетонных работ с применением электрического оборудования — лучший способ получить заводскую прочность без потери эксплуатационных качеств материала.

В зимний период бетон укрывают с целью защиты поверхности от потери тепла.

Особенности набора прочности

График твердения бетона зависит от разных факторов. При опускании температурных показателей процесс замедляется, а нулевая отметка термометра приостанавливает его, поскольку жидкость в составе начинает замерзать, а качество материала ухудшается.

При отсутствии требуемого объема влаги бетонная конструкция не может получить заводские эксплуатационные свойства, а при автоклавном отвердении процесс сильно ускоряется. Наличие влаги в воздухе сокращает интервал.

График набора прочности бетона В25 определяется его составом. Составы более высокой марки твердеют быстрее, что заставляет работников приступать к обработке более оперативно. В период с 3 по 10 сутки после заливки материалу нужно обеспечивать благоприятные условия. При теплой погоде раствор укрывают водоотталкивающей пленкой, а сам камень увлажняется каждые сутки по 6-7 раз.

Смесь нужно изолировать от прямых лучей. В зимний период бетон прогревают искусственным путем и утепляют. Для этих целей используют специальное обогревательное оборудование, препятствующее замерзанию жидкости и защищающее конструкцию от осадков. Необходимо придерживаться нормативно-безопасного срока набора, который указывается в диаграммах СНиП.

От чего зависит набор прочности

Среди ключевых факторов, влияющих на интенсивность получения прочности, выделяют:

  1. Марку цементной смеси.
  2. Пропорции воды и цемента.
  3. Пропорции других добавок.
  4. Метод уплотнения.
  5. Температурно-влажностный режим.
  6. Способ и скорость укладки.
  7. Качество и интенсивность увлажнения.

По мере повышения марки бетона нужно менять пропорции компонентов, поскольку от них зависят конечные прочностные свойства.

Фундаменты из высоких марок цементной смеси характеризуются повышенной надежностью, большим сроком службы и прочностью. В холодный период камень становится более прочным из-за способности выделять тепло, однако, чтобы сбалансировать график образования монолита, лучше внести в состав специализированные добавки. Они предназначаются для ускорения твердения и остановки гидратации.

С такими компонентами состав приобретает марочную прочность уже через 2 недели. На набор прочностных свойств влияет тип компонентов состава. Так, глиноземистый цемент может упрочняться даже в сильный мороз, поскольку он способен выделять в 7 раз больше тепла, чем классический портландцемент.

Важное значение отыгрывает форма и фракция зерен органических добавок. Если они обладают неправильной формой и шероховатой поверхностью, это создает благоприятные условия сцепления и повышает качество материала. По мере увеличения доли воды происходит расслоение массы.

Для ускорения процесса и сокращения термина выдержки бетона лучше воспользоваться пескобетонами с минимальным соотношением воды/цемента. Если материал не имеет хорошего уплотнения, в процессе созревания он получит не больше 50% от заявленной прочности. Используя ручные уплотняющие приспособления, можно поднять показатель на 30-40%.

График по суткам

График получения заводской прочности бетона по суткам указывает временной интервал, за который смесь приобретает заводские свойства. В благоприятной среде состав успевает «созреть» за 28 суток, при этом наибольшая эффективность твердения замечается в течение первых 5 дней. Через неделю с момента заливки прочностной показатель достигает 70%. При этом приступать к дальнейшим работам разрешается только после получения 100% значения, т.е. через 28 суток.

Однако при изменении окружающих условий показания графика могут меняться. Чтобы точно определить, за сколько времени бетон полностью затвердеет, следует выполнить контрольные испытания образцов.

В теплую пору процесс оптимизируется с помощью 2 методов:

  1. Выдержка бетона в опалубке.
  2. Созревание смеси после демонтажа опалубочной конструкции.

Если работа выполняется в холодный период, конструкцию нужно дополнительно обогревать и защищать гидроизолирующими материалами. В противном случае процесс полимеризации будет замедлен.

Марка бетона М200-М300 (раствор создавался на базе портландцемента М400-М500)Среднесуточная температура, при которой твердеет бетон, °CИнтервал твердения
1235714
Прочность бетона на сжатие (% от заводского значения)
-3368121520
051218283550
+591927384862
+10122537505872
+20234050657590

Для ускорения процесса и сокращения времени выдержки следует воспользоваться пескобетонами с минимальным соотношением воды к цементу. Если пропорции воды и цемента равны ¼, сроки из графика будут сокращены в 2 раза. Чтобы получить положительный результат, состав можно разбавить пластификаторами.

Нормативные документы, регламентирующие набор прочности бетонной смеси

Ключевым документом, регламентирующим сроки и условия твердения бетона, является ГОСТ 18105-2010. Еще обработка бетона контролируется стандартом ГОСТ 26633-2012. Для промышленного возведения построек используются другие правовые акты.

Прочностные свойства бетонных конструкций зависят от многих факторов и создаются под воздействием различных условий. Задача строителей заключается в подготовке правильной бетонной смеси и обеспечении благоприятных условий для повышения прочности.

График набора прочности бетона в зависимости от температуры

Самым важным показателем качества бетонов является прочность материала. Согласно требованиям ГОСТ в условиях сжатия она может варьировать в диапазоне М50-800. Наибольшей популярностью пользуются марки цемента М100-500.

Блок: 1/5 | Кол-во символов: 219
Источник: http://aquagroup.ru/articles/nabor-prochnosti-betona.html

Стадии набора прочности и влияние температуры

Вы наверняка знаете, что для достижения марочного значения бетона требуется 28 дней. Это общая цифра, которая на деле может отклоняться в большую или меньшую сторону. Чтобы возвести надежную постройку, нужно понимать сам процесс набора прочности, он состоит из двух стадий:

  • На первой стадии смесь схватывается – все компоненты бетона соединяются между собой.
  • На второй материал набирает прочность и твердеет.

Блок: 2/7 | Кол-во символов: 456
Источник: http://okbeton.ru/raschet/nabor-prochnosti-betona-v-zavisimosti-ot-temperatury.html

Срок твердения бетона

Подавляющее большинство самодеятельных строителей считают по не совсем понятным причинам, что за окончанием укладки в опалубку либо завершением работ по выравниванию стяжки процесс бетонирования законченным. Между тем, время схватывания бетона значительно больше, чем время на его укладку. Бетонная смесь – живой организм, в котором по окончании укладочных работ происходят сложные и протяженные по времени физико-химические процессы, связанные с превращением раствора в надежную основу строительных конструкций.

Прежде чем производить распалубку и наслаждаться результатами приложенных усилий, нужно создать максимально комфортные условия для созревания и оптимальной гидратации бетона, без которой невозможно достижение требуемой марочной прочности монолита. Строительные нормы и правила содержат выверенные данные, которые приведены в таблицах времени схватывания бетона.

Температура бетона, ССрок твердения бетона, сутки
12345671428
Прочность бетона, %
0202631353943466177
10273542485155597591
153039455255606481100
203443505660656987
303951576468737695
4048576470758085
5049627078849095
60546878869298
7060738496
80658092

Содержащиеся в официальных таблицах данные, конечно, должны служить ориентиром при самостоятельном обустройстве бетонных или железобетонных конструкций. Но применение таких данных должно происходить в плотной практической привязке к реальным условиям строительства.

Блок: 2/5 | Кол-во символов: 1437
Источник: https://betonshchik.ru/poleznoe/vremja-shvatyvanija-betona.html

Как бетон набирает прочность?

После укладки в смеси начинают происходить физико-химические процессы по превращению его в прочную основу для строительной конструкции. Как только под их влиянием вода и цемент вступают во взаимодействие, раствор постепенно теряет свою подвижность и изменяет свойства. Формирование новой структуры происходит в течение определенного времени. Вызревание бетона предполагает прохождение раствором двух стадий: начальной — схватывания, и завершающей — затвердевания. Их прохождение дает возможность получить прочностные свойства соответствующие бетону определенного класса и марки.

Стадия схватывания

Во время транспортировки в автобетоносмесителе смесь остается подвижной благодаря постоянному перемешиванию и тиксотропным ее свойствам. Прекращение механического воздействия на раствор после заливки увеличивает его вязкость, и он начинает схватываться. Все выявленные дефекты нужно устранять в начале первой стадии вызревания, она начинается сразу после заливки бетонной смеси и длится недолго.

Время схватывания зависит от температуры воздуха. Постоянная температура +20°С считается идеальным условием для первой стадии застывания раствора, позволяющим ему схватиться за 3 часа. При изменении этого условия длительность схватывания может уменьшиться или увеличиться. Дольше всего эта стадия длится при температурных значениях окружающего воздуха близких к 0 градусов.

Стадия твердения

После окончательного схватывания раствора начинается стадия твердения. На начальном этапе заполнитель, скрепленный кристаллизованными частицами цемента, не обеспечивает требуемую прочность. Но с началом реакции гидратации, твердение становится наиболее динамичным. Бетонная основа за 7 суток становится намного прочнее. За этот небольшой отрезок времени бетон набирает 70 процентов прочности. После происходит замедление этого процесса и еще 25% твердости набираются на протяжении трех недель. Полное затвердевание происходит через несколько лет.

Блок: 2/4 | Кол-во символов: 1955
Источник: https://betonpro100.ru/harakteristiki-i-svojstva/nabor-prochnosti-betona

Уход за бетоном после заливки: основные цели и методы

Процессы, связанные с проведением мероприятий, которые предшествуют распалубке, содержат несколько технологических приемов. Цель выполнения таких мероприятий одна – создание железобетонной конструкции, максимально соответствующей по своим физико-техническим свойствам параметрам, которые заложены в проект. Основополагающим мероприятием, безусловно, является уход за уложенной бетонной смесью.

Уход заключается в выполнении комплекса мероприятий, которые призваны создать условия, оптимально соответствующие происходящим в смеси физико-химическим преобразованиям, во время набора прочности бетона. Неукоснительное следование предписанным технологией ухода требованиям позволяет:

  • свести к минимальным значениям усадочные явления в бетонном составе пластического происхождения;
  • обеспечить прочностные и временные значения бетонного сооружения в параметрах, предусмотренных проектом;
  • предохранить бетонную смесь от температурных дисфункций;
  • препятствовать прелиминарному отвердению уложенной бетонной смеси;
  • предохранить сооружение от различного происхождения воздействий механического или химического генеза.

Процедуры ухода за свежеобустроенной железобетонной конструкцией следует начинать непосредственно по окончании укладки смеси и продолжаться до тех пор, пока ей не будет достигнуто 70 % прочности, предусмотренной проектом. Это предусматривается требованиями, изложенными в пункте 2.66 СНиПа . Распалубку можно провести и в более ранние сроки, если это обосновано сложившимися параметрическими обстоятельствами.

После окончания укладки бетонной смеси следует провести осмотр опалубочной конструкции. Цель такого осмотра – выяснение сохранения геометрических параметров, выявление протечек жидкой составляющей смеси и механических повреждений элементов опалубки. С учетом того, сколько времени застывает бетон, точнее сказать – с учетом времени его схватывания, проявившиеся дефекты необходимо устранить. Среднее время, за которое может схватиться свежеуложенная бетонная смесь, составляет около 2-х часов, в зависимости от температурных параметров и марки портландцемента. Конструкцию необходимо предохранять от любого механического воздействия в виде ударов, сотрясений, вибрационных проявлений столько, сколько времени сохнет бетон.

Блок: 3/5 | Кол-во символов: 2295
Источник: https://betonshchik.ru/poleznoe/vremja-shvatyvanija-betona.html

Факторы, влияющие на прочность

Практически все работы с раствором проводятся на открытом воздухе как летом, так и зимой. Погодные условия и температура воздуха оказывает непосредственное влияние на время застывания бетона. Таким образом, на набор прочности влияют следующие факторы:

  • температура;
  • влажность;
  • класс материала;
  • время.

Чем ниже температура на улице, тем медленнее и дольше будет происходить процесс затвердения. Зимой, в естественных условиях, эта процедура полностью останавливается, так как вода не испаряется, а замерзает. При повышении температуры застывание раствора опять продолжится. Чтобы это лучше понять, стоит обратиться к графику твердения бетона В25 или В30.

График представляет собой кривые линии, показывающие, как долго и при какой температуре достигается определенная прочность бетона. Если летом твердение бетона протекает естественным образом, то зимой необходимо принимать меры для его застывания. Для этого в бетонную смесь добавляют специальные противоморозные вещества, которые способствуют сохранению свойств приготовленного раствора.

При этом они не дают воде быстро замерзать и позволяют качественно провести заливку бетонной смеси. При более низких температурах сразу после заливки раствора обеспечивают его прогрев. Обычно для этого используют электрический ток или тепловые обогреватели. В первом случае с помощью проводов по контурам производят подключение непосредственно арматуры в опалубке или через электроды, погруженные в раствор.

Причем контуры не должны касаться друг друга, иначе будет короткое замыкание. Все подключение ведется через специальный масляный трансформатор для прогрева бетона. Во втором случае место бетонирования накрывают шатром и подключают несколько воздушных обогревателей. Большую роль играет повышенная влажность воздуха. Если ее показатели достигают 70—90%, то прочность раствора значительно увеличивается.

Блок: 3/4 | Кол-во символов: 1871
Источник: https://TvoiDvor.com/beton/grafik-nabora-prochnosti-betona-v-zavisimosti-ot-temperaturyi/

Вторая стадия

Когда первая стадия завершена, материал начинает твердеть. Необходимую прочность бетон набирает уже через четыре недели, но окончательный набор прочности завершится только через несколько лет. Марку бетона специалисты смогут определить через 28 дней. Набор прочности бетона в зависимости от влажности и температуры проходит с разной скоростью. В первые 5-6 дней после заливки процесс протекает наиболее интенсивно. После первых трех суток материал получит 30% прочности от марочного значения, которое мы узнаем только через 4 недели.

Через две недели после заливки бетон наберет до 70% прочности, а через 90-100 дней прочность превысит марочный показатель на 20%. Прекратится процесс через несколько лет, но прочность изменится незначительно. При проверке бетона, залитого 3 года назад, можно узнать, что его прочность вдвое превысила марочный показатель.

На таблице ниже показано, как длительность набора прочности зависит от температуры:

Блок: 4/7 | Кол-во символов: 960
Источник: http://okbeton.ru/raschet/nabor-prochnosti-betona-v-zavisimosti-ot-temperatury.html

Методы ускорения застывания бетона

Очень часто в процессе строительства необходимо ускорить процесс набора прочности бетона. Так, при заливке монолитных конструкций и ограничении сроков строительных работ применяют смеси на основе сернокислых, углекислых и аммонийных солей, хлоридов и нитратов кальция.

Применение этих добавок позволяет сократить длительность застывания бетона в 2 раза. Стоит заметить, что такие работы проводят в летний период и антиморозные добавки здесь не подойдут. В сильно жаркую и сухую погоду проводят увлажнение залитого раствора, так как очень быстро испаряется вода и происходит нарушение графика набора прочности материала.

Для этого верхнюю часть раствора накрывают материалом или посыпают опилками и периодически смачивают их по мере испарения воды. На асфальтобетонных заводах для ускорения застывания раствора применяют способ пропаривания. Процедуру эту проводят на открытом воздухе или в специальных закрытых камерах, где за 6—16 часов изделия из бетона набирают 60—70% прочности.

Блок: 4/4 | Кол-во символов: 1014
Источник: https://TvoiDvor.com/beton/grafik-nabora-prochnosti-betona-v-zavisimosti-ot-temperaturyi/

График набора прочности бетона

Временной интервал, на протяжении которого происходит обретение раствором необходимых эксплуатационных свойств, называется периодом выдерживания бетона, после которого можно наносить защитный слой бетона. График набора прочности  отражает время, которое требуется бетону для достижения максимального значения прочности.

В нормальных условиях состав «созревает» за 28 дней. На протяжении первых 5-ти дней происходит интенсивное твердение бетона. Спустя 7 дней после заливки достигаются 70% прочности выбранной марки. Однако дальнейшие строительные работы специалисты советуют начинать лишь при достижении 100% — не ранее, чем через 28 дней после заливки.

Время набора прочности бетона для каждого отдельного случая может несколько отличаться. Для точного определения срока твердения состава проводят контрольные испытания образцов материала.

В теплое время года в монолитном домостроении для оптимизации процесса выдерживания состава и обретения им оптимальных механических и физические свойства достаточно следующих операций:

  • Выдерживание в опалубке бетона.
  • Дозревание состава после удаления опалубки.

Если мероприятия проводятся в холодное время года, для достижения должной марочной прочности следует обеспечить дополнительное обогревание бетона и его гидроизоляцию. Связано это с тем, что при снижении температуры происходит замедление процесса полимеризации.

Чтобы ускорить набор прочности и минимизировать время выдержки бетона рекомендуется использовать пескобетоны с низким водоцементным соотношением. При соотношении вода и цемент 1/4 сроки, приведенные в таблице, сокращаются в 2 раза. Для достижения такого результата в состав добавляются пластификаторы. Также сократить срок созревания состава можно, искусственно увеличив температуру.

Блок: 2/5 | Кол-во символов: 1781
Источник: http://aquagroup.ru/articles/nabor-prochnosti-betona.html

Согласно ГОСТ

Необходимая марка и класс бетона определяется с учетом составленного проекта. Необходимые показатели прочности могут меняться в зависимости от применяемых строительных материалов. Например, при возведении дома на основе легких бетона для основания нет необходимости применять бетон высокой прочности. Когда стены строения будут выполнены из кирпича, то бетон должен иметь высокие прочностные характеристики. Например, для этого используют тяжелый и мелкозернистый бетон по стандарту 26633 ГОСТ.

Для определения прочности применяется ГОСТ 18105-86. В этом случае необходимо подготовить проект или же посмотреть информацию со схожего.

Прочность – это главный показатель качества для бетона ГОСТа любого уровня. Процесс его затвердения начинает происходить уже в первые часы после того, как соединили воду и цемент, а вот его длительность зависит от различных факторов: температуру, влажность, состав бетона. Если вес необходимые условия были соблюдены точно, то процесс набора прочности будет окончен по прошествии 28 дней, а вы сможете приступить к необходимым работам.

Блок: 5/5 | Кол-во символов: 1103
Источник: https://ResForBuild.ru/beton/rastvor/grafik-nabora-prochnosti-betona.html

Зависимость времени набора прочности от марки бетонной смеси

Логически понятно, что применение для приготовления бетонных составов разных марок портландцемента приводит к изменению времени твердения бетона. Чем выше марка портландцемента, тем меньше время для набора прочности требуется смеси. Но при использовании любой марки, будь это марка 300 либо 400, не следует прикладывать к железобетонной конструкции значительные механического характера нагрузки раньше, чем по истечении 28 дней. Хотя время схватывания бетона по таблицам, приведенным в строительных правилах, может быть и меньше. Особенно это касается бетонов, приготовленных с применением портландцемента марки 400.

Марка цементаВремя твердения различных марок бетона
за 14 сутокза 28 суток
100150100150200250300400
3000.650.60.750.650.550.50.4
4000.750.650.850.750.630.560.50.4
5000.850.750.850.710.640.60.46
6000.90.80.950.750.680.630.5

Проектирование, строительство и окончательное обустройство любых построек с применением железобетонных компонентов требует внимательного отношения ко всем стадиям возведения. Но от тщательности изготовления бетонных составляющих, в особенности фундаментов, в значительной степени зависит долговечность и надежность всего сооружения. Соблюдение сроков, за какое время схватываются бетонные смеси и составы, можно с уверенностью назвать основой успеха в любом строительном процессе.

Блок: 5/5 | Кол-во символов: 1416
Источник: https://betonshchik.ru/poleznoe/vremja-shvatyvanija-betona.html

Вывод

В реальности прочностные показатели бетонных конструкций могут изменяться по очень многим причинам. Важно обеспечить оптимальные параметры для реализации по времени графика роста прочностных свойств, соответствующих марке бетона.

Блок: 6/6 | Кол-во символов: 234
Источник: https://kladembeton.ru/poleznoe/nabor-prochnosti-betona.html

Заключение

Как показывает практика, существует множество причин изменения прочностных показателей бетона. Важно учитывать пропорции, качество компонентов, особенности местности и, конечно же, температуру.

Блок: 7/7 | Кол-во символов: 231
Источник: http://okbeton.ru/raschet/nabor-prochnosti-betona-v-zavisimosti-ot-temperatury.html

Кол-во блоков: 18 | Общее кол-во символов: 20078
Количество использованных доноров: 7
Информация по каждому донору:

  1. https://TvoiDvor.com/beton/grafik-nabora-prochnosti-betona-v-zavisimosti-ot-temperaturyi/: использовано 2 блоков из 4, кол-во символов 2885 (14%)
  2. http://okbeton.ru/raschet/nabor-prochnosti-betona-v-zavisimosti-ot-temperatury.html: использовано 3 блоков из 7, кол-во символов 1647 (8%)
  3. https://betonpro100.ru/harakteristiki-i-svojstva/nabor-prochnosti-betona: использовано 1 блоков из 4, кол-во символов 1955 (10%)
  4. http://aquagroup.ru/articles/nabor-prochnosti-betona.html: использовано 5 блоков из 5, кол-во символов 4054 (20%)
  5. https://ResForBuild.ru/beton/rastvor/grafik-nabora-prochnosti-betona.html: использовано 3 блоков из 5, кол-во символов 4155 (21%)
  6. https://kladembeton.ru/poleznoe/nabor-prochnosti-betona.html: использовано 1 блоков из 6, кол-во символов 234 (1%)
  7. https://betonshchik.ru/poleznoe/vremja-shvatyvanija-betona.html: использовано 3 блоков из 5, кол-во символов 5148 (26%)

Строительный миф №2. Нужно ли после заливки бетона ждать 28 суток?

Вопрос: сколько нужно ждать, пока произойдет затвердения бетона? Как и за какое время бетон набирает прочность? Действительно ли нужно ждать 28 суток после того, как залит бетон? Когда можно нагружать бетонные конструкции?

Каждому застройщику или строителю выгоднее построить конструкцию, здание или сооружение за кратчайшие сроки. Но бытует целый ряд мнений о том, что необходимо после выполнения работ по бетонированию конструкций ждать пока конструкция «затвердеет», чтоб потом приступить к следующему этапу строительства.

Как и за какое время бетон набирает прочность?

Нужно ли после заливки бетона ожидать 28 суток?

Для правильного вывода необходимо проанализировать нормативные документы и определить режим, этапы и сроки строительства.

При выполнении бетонных работ сталкиваются с двумя актуальными вопросами:

  1. Через какое время можно снимать опалубку?
  2. Через какое время можно нагружать железобетонный элемент или конструкцию?

Рассмотрим последовательно эти вопросы.

Для сборных железобетонных изделий очень важно определить отпускную прочность.

Отпускная прочность – это набранная прочность бетона, устанавливаемая нормативами, при которой железобетонное изделие возможно поставлять с завода на строительную площадку.

Величина отпускной прочности устанавливается согласно ГОСТов или других нормативных документов в зависимости от:

  • вида и размера конструкции;
  • состава бетона;
  • условий твердения;
  • температуры окружающей среды и климатических условий региона;
  • сроком и величины загрузки;
  • условия транспортировки.

Ниже, в таблице 1 приводятся в зависимости от вида и класса бетона, усредненные значения отпускной прочности в процентах от проектной.

Таблица 1

Вид бетонаОтпускная прочность (% от проектного класса бетона)
Тяжелый бетон и бетон на пористом заполнителе с классом С10 и выше50 %
Тяжелый бетон класса С7,5 и ниже70 %
Бетон на пористом заполнителе, класс С7,5 и ниже80 %
Бетон всех видов и классов при автоклавном твердении100 %

Итак, отпускная прочность сборных железобетонных изделий в зависимости от целого ряда факторов составляет 50÷100% от проектной. Вывод №1: при достижении отпускной прочности можно уже производить монтаж и затем нагружать железобетонные конструкции, с расчетом на то, что полное нагружение (100%) наступит не позже 28 суток от момента изготовления изделий. Более конкретный порядок и сроки нагружения сборных конструкций оговаривается в ППР (проект производства работ).

Также в строительстве существует такое понятие, как распалубочная прочность.

Распалубочная прочность – это минимальная набранная прочность бетона, при которой возможно извлечь опалубку, не повреждая бетон. Для сборных железобетонных изделий опалубочная прочность должна быть достаточная для безопасной транспортировки. Условия и скорость набора прочности для каждого изделия или конструкции определяются предприятием-изготовителем.

В условиях стройплощадки, при изготовлении монолитных конструкций распалубку, как правило выполняют непосредственно перед началом загружения конструкции.

СНиП 3.03.01-87 устанавливает следующие условия распалубки железобетонных конструкций ( смотри таблицу 2).

Таблица 2

ПараметрРаспалубочная прочность (% от нормативной, на 28 сут)
Прочность бетона (в момент распалубки конструкций), не ниже:
— теплоизоляционного0,5 МПа
— конструкционно-теплоизоляционного1,5 МПа
— армированного3,5 МПа, но не менее 50 % проектной прочности
— предварительно напряженного14,0 МПа, но не менее 70 % проектной прочности
Распалубка железобетонных конструкций с последующей обработкой бетона (п. 2.34)70 % от проектной прочности

Российский нормативный документ ТР 80-98 «Технические рекомендации по технологии бетонирования безобогревным способом монолитных конструкций с применением термоса и ускоренного термоса» приводит следующие разрешения по распалубки и нагрузки конструкций, таблица 3.

 Необходимая прочность бетона для распалубки и нагрузки конструкции:

Таблица 3

Строительные конструкцииФактическая нагрузка, % от нормативной
свыше 70%70% и менее
прочность бетона, % от проектной
Боковые щиты опалубки на фундаменте и колоннах, стенах, ригелей и балок допускается при нормальных условиях тверденияСнимать через 6 — 72 ч
Несущие щиты опалубки100См. ниже
Длина пролета несущих железобетонных плит до 3 м10070
Длина пролета несущих железобетонных плит (кроме плит) до 6 м10070
Колонны, несущие конструкции (балки, ригели, плиты) пролетом 6 м и более10080
Конструкции с напрягаемой арматурой10080

Примечания:

  1. Следует твердо помнить, что полностью на 100 % загружать конструкцию можно только, когда бетон наберет свою полную проектную прочность.
  2. Снимать боковые щиты ненесущей части опалубки можно при условии, когда разность температур между бетоном и наружным воздухом соответствует следующему условию:
  • Dt = 20 °С для конструкций с Мп = 2 – 5;
  • Dt = 30 °С для конструкций с Мп больше 5, где Мп — модуль поверхности конструкции (отношение суммы площадей охлаждаемых поверхностей конструкций в м2 к ее объему в м3), м-1 .

Дальнейшие мероприятия по выполнению опалубочных работ и движение работников по железобетонным конструкциям допускается, когда прочность бетона составляет 1,5 МПа и более. (СНиП 3.03.01-87, п. 2.17). Также, в этом нормативном документе есть указание (п.2.110), что при использовании промежуточных опор (подпорок) для перекрытия пролетов, при частичной или последовательной снятии опалубки, допустимая распалубочная прочность может быть понижена, а это означает большую оборачиваемость опалубки и уменьшения сроков строительства. Более конкретные мероприятия по раннем снятие опалубки должно определятся исходя из конкретных условий строительства и освещаться в ППР.

Некоторые литературные источники указывают следующие значения для распалубки железобетонных конструкций, табл. 4:

 Таблица 4

КонструкцияМинимальная распалубочная прочность (% от нормативной, на 28 сут)
Железобетонные плиты и своды с длиной пролета до 2 м50%
Железобетонные балки с длиной пролета до 8 м70%
Все несущие железобетонные конструкции с длиной пролета более 8 м100%
Железобетонные конструкции с жесткой арматурой (колоны, армированные сварными несущими двутавровыми балками)25%

Вывод №2: исходя из всего выше приведенного и анализируя все таблицы по распалубочной прочности бетона и его нагружении, распалубочная прочность находится в пределах 50…80% от проектной. Тогда:

  1. распалубку конструкции допускается проводить, когда фактическая прочность бетона достигнет 70% от проектной, и в этом случае можно постепенно загружать дальше;
  2. распалубку конструкции допускается проводить, при фактической прочности 50% от проектной, только необходимо установить дополнительные опоры для страховки и исключения прогибов. В этом случае также можно постепенно нагружать конструкцию (ставить опалубку, кладку, и т.д.).
Через сколько времени бетон может набрать распалубочную прочность, при которой можно еще и нагружать конструкцию?

Как уже выше вспоминалось, при разных условиях (температура, влажность, атмосферные осадки и т.д.) разный бетон набирают прочность по разному. На рис. 2 приведен график скорости набора прочности в зависимости от температуры ТВО (тепло влажностной обработки).

Из графика видно, что в лабораторных условиях при постоянной температуре 60°С среднюю распалубочную прочность бетон (70%) приобретает через 32 часа (1,3 сут), а при температуре 30°С – приобретает примерно за 4 сут.

Так как на строительных объектах, в течении суток температура окружающего воздуха колеблется, то берут во внимание среднесуточную температуру, которая летом составляет 18…28°С, а осенью достигает и 5…10°С. При таких температурах бетон будет набирать прочность намного медленнее.

Рис. 1. График скорости набора прочности бетона в зависимости от температуры ТВО (тепло влажностной обработки) [1]

На предприятиях по изготовлению бетона и конструкций из него, должны быть графики набора прочности бетона определенного состава. Для примерного определения прочности конкретного бетона, можно воспользоваться графиками набора прочности в зависимости от вида цемента, температуры и класса бетона (рис. 2) из нормативных документов [2, 3].

Ниже приведен рост прочности бетона в зависимости от температуры окружающего воздуха или ТВО, (в % от R28):

а) класс С15–С25 на основе портландцемента марки М400

б) класс С30 на основе портландцемента марки М500

в) класс С15–С25 на основе шлакопортландцемента марки М400

г) класс С40 на основе портландцемента марки М600

д) быстротвердеющий высокоактивный портландцемент (БТЦ)

Графики набора прочности (табл. 5-9)

Набор прочности бетона класса С15 – С25 на портландцементе марки М400 (% от R28):

Таблица 5

Возраст бетона, сут.Температура бетона, °С
-305102030405060
1/21451217283850
1359122335455563
261219254055657580
3818273750657785
512283850657890
715354858758798
142050627287100
2825657785100

Набор прочности бетона класса С30 на портландцементе марки М500 (% от R28):

Таблица 6

Возраст бетона, сут.Температура бетона, °С
-305102030405060
1812182840556570
21622325063758590
3102232456074859298
516324558748596
7194055668292100
142557708092100
2830709090100

Набор прочности бетона класса С15 – С25 на шлакопортландцементе марки М400 (% от R28):

Таблица 7

Возраст бетона, сут.Температура бетона, °С
-305102030405060
1/224720253242
136101630405065
23812183040607590
3513182540557090
58202735556585
710253443657092
14123550608096100
2815156580100

Набор прочности бетона класса С40 на портландцементе марки М600 (% от R28):

Таблица 8

Возраст бетона, сутТемпература бетона, °С
0510203040
181321324559
2172536526575
3233546627483
7425768839098
1458738294100
28718392100

Набор прочности бетона с применением противоморозных добавок:

Таблица 9

Противоморозная добавкаВид вяжущегоТемпература твердения бетона, °СПрочность бетона, % от R28 при твердении на морозе через число суток
7142890
1) Нитрит натрия (в водном растворе), NaNO2портландцемент-5254060100
-1015253570
-155102050
2) Нитрит натрия кристаллический, NaNO2портландцемент-5254060100
-1015253570
-155102050
3) Нитродапшлакопортландцемент-515254590
-1010152560
-1551540

Вывод №3: из графиков и таблиц видно, что бетон на основе портландцемента при среднесуточной температуре 10 и выше набирает 50% прочности от проектной за 5…7 суток, а бетон на шлакопортландцементе набирает при тех же самых условиях – за 14 и более суток. Зимой при отрицательных температурах с применением даже противоморозных добавок (табл.9) бетон набирает проектную прочность за 90 суток и больше. Для ускорения времени набора требуемой прочности при зимнем бетонировании необходимо использовать электропрогрев.

Для быстрого набора прочности, согласно СНиП 3.03.01-87 «Несущие и ограждающие конструкции. 2. Бетонные работы» (п. 2.15) за бетоном нужен соответствующий уход. Уход за бетоном начинается сразу после укладки его в опалубку и продолжают до момента распалубки. Бетон следует хранить от прямого попадания солнечных лучей и атмосферных осадков, ветра, а также создать тепловлажностные условия для его твердения (накрыть пленкой). Рекомендуется бетон изготовленный на портландцементе в течении 7 суток поливать водой, а на основе малоактивных и шлакопортландцементах поливать не менее 14 суток. При температуре воздуха 15°С рекомендуется поливать бетон через 3 часа в течении первых 3 суток. При средней температуре воздуха от +5 до 0°С полив и смачивания бетона не осуществляется. Полная нагрузка (расчетная) железобетонных конструкций допускается только после того, как бетон будет иметь проектную прочность.

Рекомендации по выполнению фундаментов

Отдельно хотелось заострить внимание на фундаменте, так как есть некоторые особенности его работы:

  1. Наилучшее время для строительства фундамента является лето (хороший температурный режим).
  2. Нежелательно, подвергать фундамент длительному простою, т.к. замокание котлована, морозное пучение, попеременное замораживание и оттаивание грунтов основания приводит к его разрушению.
  3. Выше перечисленные факторы приводят к неравномерной усадке фундамента.
  4. Если все-таки есть необходимость оставить фундамент зимовать, необходимо его «законсервировать» — закрыть и защитить от атмосферных осадков, исключить замокания и затопление грунта вблизи фундамента (примерно 0,4…0,5 м).
  5. Так как бетон при благоприятных условиях набирает 50…80% от проектной прочности за 7…14 дней, тогда допускается нагружать фундамент через 7…14 суток, с учетом, что полное нагружение (100%) наступит только после 28 суток с момента заливки фундамента.
  6. При использовании ускорителей твердения при нормальной температуре возможно уже нагружать фундамент и через 5 дней.
  7. Фундамент следует нагружать равномерно, чтобы избежать неравномерной осадки основания.

Для более точной подстраховки для контроля прочности фундаментов или других железобетонных конструкций изготавливают серию стандартных образцов-кубов 150х150х150 или 100х100х100 мм, которые потом испытывают на сжатие.

Литература:

  1.  Как построить дом. Как бетон набирает крепость? Время затвердевания бетона, график набора крепости. Режим доступа: ссылка на статью.
  2. ТР 80-98 Технические рекомендации по технологии бетонирования безобогревным способом монолитных конструкций с применением термоса и ускоренного термоса. МОСКВА – 1998.
  3. ВСН 20-68 Указания на бетонирование в зимнее время дорожных оснований под асфальтобетонные покрытия в г. Москве.

 Автор публикации эксперт GIDproekt

Конев Александр Анатольевич

 

 

Как оценить прочность бетона на месте

Бетон должен набрать достаточную прочность, чтобы выдерживать свой вес и строительные нагрузки, прежде чем снимать опалубку, перекладывать шоры или задвигать. Инженеры часто определяют минимальную прочность бетона на месте, прежде чем подрядчики смогут выполнить последующее натяжение, засыпать стены, открыть тротуары для движения или прекратить защиту в холодную погоду. По этим причинам подрядчики должны знать, как правильно оценить прочность бетона на месте для недавно уложенного бетона, особенно в холодную погоду.В противном случае безопасность рабочих и качество конструкции могут быть поставлены под угрозу.

Испытательные цилиндры для испытаний в полевых условиях и коэффициенты зрелости часто используются для оценки прочности бетона на месте. Однако испытание цилиндров, отвержденных в полевых условиях, является стандартной процедурой, установленной строительными нормами. Другие методы — включая факторы зрелости и монолитные цилиндры для плит, сопротивление проникновению и прочность на вырыв — требуют одобрения архитектора / инженера и могут потребовать одобрения строительного чиновника.

Температура и время

Прирост прочности бетона зависит от комбинации температуры и времени выдержки. Скорость гидратации или химической реакции между цементом и водой зависит от температуры бетона. По мере повышения температуры бетона скорость гидратации и, как следствие, увеличение прочности увеличивается. И наоборот, скорость набора прочности снижается с понижением температуры бетона. По этой причине замедленная прочность бетона является обычным явлением в холодную погоду, если подрядчики не соблюдают меры предосторожности.Конечно, прочность бетона со временем увеличивается, если есть адекватные условия отверждения, способствующие гидратации.

Полевые испытательные цилиндры

Стандартное и полевое отверждение — это разные процедуры, определенные ASTM C31 для отверждения бетонных испытательных цилиндров. Испытательные цилиндры стандартного отверждения, иногда называемые цилиндрами лабораторного отверждения, представляют собой идеальную или номинальную прочность бетона. Диапазон температур для стандартного отверждения составляет от 60 ° F до 80 ° F в течение периода до 48 часов (начальное отверждение) и 73.5 ± 3,5 ° F для баланса 28-дневного периода отверждения (окончательное отверждение) для бетонов с указанной прочностью до 6000 фунтов на квадратный дюйм. Бетон с указанной прочностью 6000 фунтов на квадратный дюйм или выше должен соответствовать более жесткому диапазону температур от 68 ° F до 78 ° F для начального отверждения. Для стандартного отверждения температура и время стандартизированы для обеспечения однородных условий отверждения. Вот почему значения прочности, полученные из испытательных цилиндров стандартного отверждения, используются для определения прочности бетона.

Полевое отверждение отличается от стандартного.Он заключается в хранении испытательных цилиндров как можно ближе к бетону на месте и защите цилиндров от элементов таким же образом, как и бетон на месте. Условия отверждения испытательных цилиндров должны быть такими же, как и условия отверждения монолитного бетона. Подвергая испытательные цилиндры той же зависимости температуры от времени, что и бетон на месте, предполагается, что прочность испытательных цилиндров представляет собой прочность бетона на месте.

Испытательные цилиндры, отверждаемые в полевых условиях, обычно недооценивают истинную прочность бетона на месте из-за тепловой массы испытательного цилиндра (4 дюйма.x 8 дюймов или 6 дюймов x 12 дюймов) по сравнению со значительно большей тепловой массой представленного бетонного элемента. Как правило, температуры отверждения для испытательных цилиндров ниже, чем фактические температуры бетона на месте, даже когда испытательные цилиндры заправлены под отверждаемое одеяло и хранятся рядом с представленным бетоном.

Значения прочности, полученные на испытательных цилиндрах, отвержденных в полевых условиях, обычно являются консервативными. Тем не менее, отвержденные в полевых условиях цилиндры могут сильно завышать прочность бетона на месте, если отвержденные в полевых условиях цилиндры хранятся и отверждаются в рабочем прицепе.

За некоторыми исключениями, прочность цилиндров стандартного отверждения выше, чем прочность цилиндров, отвержденных в полевых условиях, потому что стандартные температуры отверждения создают более высокие скорости гидратации и увеличения прочности, чем при типичных температурах отверждения в полевых условиях. По этой причине всегда используйте цилиндры стандартной прочности для определения прочности бетона. Что еще более важно, используйте только прочность цилиндров, отверждаемых в полевых условиях, для принятия конструктивных решений, таких как определение того, когда следует снимать опалубку и опоры, начинать последующее натяжение или определять, когда вводить конструкцию в эксплуатацию.Никогда не используйте испытательные цилиндры стандартного отверждения вместо испытательных цилиндров, отвержденных в полевых условиях. Неспособность правильно оценить прочность бетона на месте может поставить под угрозу безопасность рабочих и повредить конструкцию.

Метод погашения

Метод зрелости (ASTM C1074) является более точным, надежным и экономичным для оценки прочности бетона на месте, чем испытательные цилиндры, отверждаемые в полевых условиях. Он основан на концепции, согласно которой температура и время отверждения бетона напрямую связаны с прочностью бетона.В частности, этот метод использует заранее установленное соотношение температура-время-прочность для данной бетонной смеси для оценки прочности бетона на месте.

Шаги по оценке прочности бетона на месте с использованием метода зрелости включают:

1. Подготовьте не менее 15 цилиндров для лабораторных испытаний и вставьте датчики температуры по крайней мере в два из цилиндров для данной бетонной смеси, отвердите при комнатной температуре и вычислите коэффициенты зрелости M (t) для различного прошедшего времени, соответствующего испытаниям на прочность с использованием следующее уравнение:

M (t) = СУММ (Ta минус To) Δt

где:

M (t) = коэффициент зрелости в возрасте (t), градусы – часы, ° F – ч

Δt = временной интервал, час

Ta = средняя температура бетона за интервал времени (Δt), ° F

To = температура, ниже которой не происходит увеличения прочности, ° F (от 14 ° F до 32 ° F)

Затем создайте гладкую кривую зависимости прочности от зрелости, построив рассчитанные коэффициенты зрелости M (t) в зависимости от соответствующей прочности бетона.

2. Измерьте зависимость температуры и времени бетона на месте путем встраивания датчиков температуры в критические места в зависимости от степени воздействия бетона и условий нагрузки.

3. Считайте данные температура-время и рассчитайте коэффициент зрелости для прошедшего времени бетона на месте, используя уравнение коэффициента зрелости M (t). Современное оборудование для погашения автоматически рассчитывает и записывает коэффициенты погашения.

4. Оцените прочность бетона на месте, введя предварительно установленную кривую зависимости прочности от зрелости с вычисленным M (t) для бетона на месте и считайте расчетную прочность, как показано на Рисунке 1.Опять же, этот шаг обычно выполняется автоматически с помощью современного современного оборудования и программного обеспечения.

Пример

Из-за приближения холодного фронта подрядчик установил датчики температуры в стене, размещенные в 9:00 1 сентября. Поставщик бетона предоставил кривую зависимости зрелости от прочности для используемого бетона, как показано на Рисунке 1. Технические характеристики для Проект требовал минимальной прочности бетона 3000 фунтов на квадратный дюйм перед укладкой и уплотнением засыпки у стены.

Как показано в таблице 1, истекшее время и температура бетона на месте были записаны в столбцах 2 и 3 для дат, указанных в столбце 1. Используя столбец 3, средние температуры бетона на месте были вычислены и занесены в столбец 4. Затем, подрядчик вычел 23 ° F, или температуру, при которой рост прочности практически прекращается, из средних температур, показанных в столбце 4, и ввел скорректированные температуры в столбец 5. Истекшее время в часах из столбца 2 было вычислено и введено в столбец 6.Затем подрядчик умножил температуры в столбце 5 на истекшее время в столбце 6 и ввел значения (° F-h) в столбец 7. Для столбца 8 были вычислены совокупные коэффициенты зрелости и введены для различных прошедших периодов времени.

Наконец, подрядчик ввел предварительно установленную кривую зависимости прочности от зрелости (рис. 1), предоставленную поставщиком бетона с учетом совокупных коэффициентов зрелости на месте из столбца 8, и прочитал соответствующие значения прочности бетона на месте.Расчетная прочность бетона на месте была введена в столбец 9 (например, для коэффициента зрелости 5070 ° F-ч соответствующая прочность бетона составила 3100 фунтов на квадратный дюйм из Рисунка 1).

Поскольку спецификации требовали прочности бетона не менее 3000 фунтов на квадратный дюйм для обеспечения достаточной прочности стены для установки засыпки, подрядчик должен подождать, пока бетон не достигнет коэффициента зрелости не менее 5000 ° F в час. Чтобы сократить период отверждения, подрядчик может использовать горячую воду для замеса, добавить химически ускоряющую добавку к бетону или добавить дополнительные теплоизоляционные покрытия, чтобы можно было генерировать и поддерживать больше тепла.

Ограничения

Ошибочные оценки прочности могут произойти, если бетон на месте значительно отличается от бетона, используемого для построения предварительно установленной кривой зависимости температуры от времени и прочности. Изменения в материалах, содержании воды и воздуха, а также в точности дозирования могут привести к ошибкам при оценке прочности. ASTM C1074 рекомендует проводить дополнительные испытания для периодической проверки кривой зависимости температуры от времени и прочности, особенно когда опасные для жизни строительные работы основаны на расчетной прочности бетона на месте.

Ссылки
ACI306R-10 Руководство по бетонированию в холодную погоду, Американский институт бетона, www.concrete.org, Mindness, S., Young, J.F, and Darwin, D., Concrete, 2nd Edition, Prentice Hall, 2003.

Ким Башам, PhD, P.E. FACI является президентом компании KB Engineering LLC, которая предоставляет инженерные и научные услуги бетонной промышленности. Бэшем также проводит семинары и тренинги, посвященные всем аспектам бетонных технологий, строительства и устранения неисправностей.С ним можно связаться по электронной почте [email protected].

Вот несколько альтернатив испытательным цилиндрам, отверждаемым в полевых условиях, для оценки прочности бетона на месте.

ASTM C31 / C31M-12 Стандартная практика изготовления и отверждения бетонных образцов для испытаний в полевых условиях — Описано в этой статье.

ASTM C873 / C873M-10a Стандартный метод испытаний прочности на сжатие бетонных цилиндров, отлитых на месте в цилиндрических формах. — Включает в себя заливку на месте испытательных цилиндров в плиты, только с глубиной от 5 до 12 дюймов.

ASTM C803 / C803M-03 (2010) Стандартный метод испытаний на сопротивление проникновению затвердевшего бетона — Включает в себя ввод штифтов в бетон с помощью инструмента с механическим приводом и проникновение измерительного штифта.

ASTM C900-06 Стандартный метод испытания прочности на вырыв затвердевшего бетона — требует установки болтов в опалубку перед укладкой бетона.

ASTM C1074-11 Стандартная практика для оценки прочности бетона по методу зрелости — Описано в этой статье.

(PDF) Влияние возраста на результаты прочности сердечника бетона

Влияние возраста на прочность сердечника бетона

16

РИСУНОК (3) ПРОЧНОСТЬ СЕРДЕЧНИКА ДО 28 ДНЕЙ СЖАТИЯ

ПРОЧНОСТЬ

ДЛЯ ВСЕХ СМЕСЕЙ В РАЗНЫХ ВОЗРАСТАХ.

IV. ЗАКЛЮЧЕНИЕ

По результатам этого исследования выводы о

можно сделать ниже:

1.Прочность всех бетонных смесей, испытанных разрушающими или неразрушающими методами

, увеличивается с возрастом при уменьшении показателя

.

2. Водоцементное соотношение бетонной смеси является значимым фактором

, который влияет на скорость набора прочности с возрастом

. Бетонные смеси с низким водоцементным соотношением достигли долгосрочных значений

быстрее, чем бетонные смеси с высоким водоцементным соотношением

. Более высокое водоцементное соотношение и

выше в разработке выше нормированных 28-дневных значений

.Развитие прочности на сжатие для смеси с

w / c 0,35 (C55) и w / c 0,55 (C25) в возрасте 120 дней составляет 95%,

и 90% соответственно, выраженное в процентах от 270

дней. соответствующие значения. Развитие прочности на сжатие

для тех же смесей, указанных выше, в возрасте 120 дней составляет 17%,

и 87% соответственно, когда выражается в процентах от соответствующих значений 28

дней.

3. Прочность на сжатие сердцевины увеличивается с возрастом бетона

, но прочность сердцевины несколько выше

, чем 28-дневная прочность на сжатие куба даже до возраста 270

дней в бетоне средней плотности (C25), в то время как прочность сердечника на сжатие

остается ниже, чем прочность на сжатие 28-дневного куба

на более высоком уровне прочности (C40 и C55) даже до возраста

270 дней.Это означает, что на прочность сердечника влияют другие факторы

(уровень прочности, условия отверждения и влажность

условий испытаний) больше, чем возраст сердечника. Таким образом, при оценке прочности сердечника

не следует использовать поправку на возраст

, особенно при отсутствии определенного влажного отверждения.

СПРАВОЧНАЯ ИНФОРМАЦИЯ

[1] Невилл А. М., «Свойства бетона», четвертый и последний

Стандарты издания

, обновленные до 2002 г., Pearson, 2005 844 p, напечатано

в Малайзии.

[2] Невилл А.М., «По моему мнению, основные тесты: легко выполнить,

нелегко интерпретировать», Concrete International, Том 23, №11,

ноябрь 2001 г., стр. 59-68 .

[3] Банджи, Дж. Х., Миллард, С. Г., Грантам, М. Г., «Тестирование бетона

в конструкциях», Тейлор и Фрэнсис, 4-е издание, 2006 г.,

, стр. 35-139.

[4] ACI 214.4R-03, «Руководство по получению стержней и интерпретации результатов прочности на сжатие

», Отчет Комитета 214 ACI,

Руководство ACI по бетонной практике, Американский институт бетона,

2004,16 стр.

[5] ASTM C42-03, «Стандартный метод испытаний для получения и испытания

просверленных стержней и распиленных балок из бетона», Ежегодная книга

стандартов ASTM Американского общества испытаний и материалов,

Том 4.2, 2006 , 5 стр.

[6] Тапкин, С., Ариоз, О., Тункан, М., Тункан, А. и Рамьяр, К.,

«Использование нейронных сетей для оценки прочности бетонного ядра

», 4-й факультет архитектуры и инженерии

Международный симпозиум, Европейский университет Лефке, Турция,

2006, стр.195-202.

[7] ACI 318M-08, «Требования к строительным нормам для конструкционного бетона

», Отчет Комитета 318 ACI, Руководство ACI по

Практика бетона, Американский институт бетона, 2008 г.

[8] ACI 301M-99 , «Спецификации конструкционного бетона»,

Сообщено Комитетом 301 ACI, Руководство ACI по бетону

Практика, Американский институт бетона, 2004 г., стр.9.

[9] BS1881: часть 6089: 1981, «Руководство по оценке прочности бетона

в существующих конструкциях», Британский институт стандартов,

1999, 15 стр.

[10] Бартлетт, Ф. М., МакГрегор, Дж. Г., «Прочность бетона

, установленная эквивалентным

на основе данных испытаний ядра», Concrete International,

Vol. 17, No. 3, март 1995 г., стр. 52–58.

[11] BS1881: part120: 1983, «Метод определения прочности бетонных стержней на сжатие

», British Standards

Institution, 1983, 5p.

[12] Шетти, М.С., «Технология, теория и практика бетона», 6-е разноцветное издание

, 2009 г., стр.428.

[13] Бартлетт, Ф. М., МакГрегор, Дж. Г., «Влияние влажности

на прочность бетонного сердечника», журнал ACI Materials Journal,

Vol. 91, № 3, май-июнь 1994 г., стр. 227-236.

[14] Бартлетт, Ф. М., и МакГрегор, Дж. Г., «Сердечники из бетонных балок с высокими характеристиками

», ACI Materials Journal, Vol. 91,

No. 6, ноябрь-декабрь 1994 г., стр. 567-576.

Неразрушающий контроль бетона: базовое руководство

Быстрый поиск в Интернете покажет, что существует множество методов неразрушающего контроля, из которых можно выбрать.Имея такое количество доступных тестов, как узнать, какой метод и оборудование подходят вам? В этом сообщении блога будет рассказано о важности неразрушающего контроля, о восьми основных методах, о том, когда их следует использовать и какое оборудование вам нужно.

В чем важность неразрушающего контроля?

Испытание затвердевшего бетона на месте часто необходимо для определения пригодности конструкции для предполагаемого использования. Методы неразрушающего контроля используются для оценки свойств бетона путем оценки прочности и других свойств, таких как коррозия арматуры, проницаемость, растрескивание и структура пустот.Этот тип тестирования важен для оценки как новых, так и старых конструкций. Основные области применения новых конструкций в основном используются для определения качества материалов. Тестирование существующих конструкций обычно связано с оценкой структурной целостности.

Преимущества неразрушающего контроля

Неразрушающий контроль также может использоваться в качестве начального шага для последующего отбора керна и более инвазивных мер, таких как:

  • Измерение характеристик предварительно отлитых, монолитных или строительство на месте
  • Определение приемлемости поставляемого материала и компонентов
  • Определение местоположения и категоризация трещин, пустот, сот и других дефектов в бетонной конструкции
  • Определение однородности бетона перед резкой стержня, испытанием под нагрузкой или другими более дорогостоящими или разрушающие испытания
  • Мониторинг развития прочности, связанного с снятием опалубки, прекращением отверждения и приложением нагрузки
  • Определение положения, количества или состояния арматуры
  • Подтверждение или обнаружение предполагаемого разрушения бетона в результате таких факторов, как перегрузка, усталость, внешнее или внутреннее химическое воздействие или изменение, пожар, взрыв, воздействие на окружающую среду ects
  • Оценка потенциальной прочности бетона при мониторинге долгосрочных изменений свойств

Методы неразрушающего контроля

  1. Windsor Probe — Предлагает быстрое и точное определение прочности бетона на сжатие.В этом методе используется зонд из закаленной стали, управляемый пороховым зарядом, который проникает через поверхность бетона. Производитель предоставляет диаграмму твердости по Моосу для заполнителя в зависимости от глубины проникновения, чтобы определить прочность бетона.
  • Когда использовать Windsor Probe — полезная опция для оценки прочности бетона на сжатие для общей оценки качества бетона и относительной прочности в различных частях конструкции.Он достаточно мал для использования в полевых условиях, а работа проста и требует небольшого обучения.
  • Молоток для испытаний бетона — Используется для оценки прочности и однородности бетона на месте, а также для определения областей плохого качества или разрушенного бетона. Подпружиненный молоток ударяется о поршень, находящийся в контакте с бетонной поверхностью, а индикатор скольжения фиксирует расстояние, на которое он отскакивает, по линейной шкале. Затем число отскока связывается с фактическим значением прочности на сжатие, чтобы установить относительную и пропорциональную прочность одной и той же бетонной смеси между различными участками конструкции.
    • Когда использовать Испытательный молоток лучше всего использовать для определения профиля относительной прочности конструкции. В идеале с помощью этого прибора один техник может быстро проанализировать большие площади с потенциальными проблемами прочности и сузить конкретные области для более тщательного тестирования. Затем области с более низким числом отскока можно экономически оценить с помощью кернов, испытаний на проникновение или измерения скорости импульса, в то время как области с более высокой прочностью можно обойти.
  • Coring — Это обычная и наиболее широко распространенная практика извлечения образцов из затвердевшего бетона для прямого определения прочности.Хотя технически это «разрушающий» метод, при осторожном использовании керны часто можно извлечь из мест, которые не повлияют на целостность конструкции. Образцы керна предлагают наиболее точные результаты для определения прочности на сжатие любым из перечисленных здесь методов, но они могут привести к косметическим повреждениям и являются трудоемкими для извлечения.
    • Когда использовать — Удаление керна часто является конечным результатом программы оценки, которая начинается с использования молотков для испытаний бетона, пробников Windsor или других неразрушающих методов.Сердечники часто считаются последним словом в определении прочности затвердевшего бетона.
  • Испытание бетона на зрелость — Бетон со временем набирает прочность и выделяет тепло при отверждении. Регистрация температуры монолитного бетона с течением времени с последующим применением стандартных математических уравнений к данным позволяет оператору установить корреляцию с лабораторными образцами известной прочности. Измерители зрелости бетона собирают значения температуры от датчиков, погруженных в свежий бетон, и регистрируют их вместе с собранным временем.На основе этих данных рассчитывается значение либо эквивалентного возраста, либо температурно-временного фактора и используется для оценки прочности на сжатие.
    • Когда использовать Испытание на зрелость — хороший вариант, если вам нужен простой и надежный способ оценки прочности бетона в раннем возрасте для безопасного снятия опалубки и сокращения задержек при укладке тротуаров и конструкции в эксплуатацию.
  • Мониторы трещин в бетоне — Измерьте ширину трещин в бетонных конструкциях, таких как мосты, здания и дороги.Перекрывающиеся верхняя и нижняя пластины отмечены, и открытие и закрытие трещины можно контролировать по приращениям.
    • Когда использовать Мониторы трещин можно использовать для периодического измерения трещин в полевых условиях для простого и точного определения движения структурного фундамента.
  • Испытание на влагостойкость — Ежегодно в результате миграции влаги через бетонные плиты и конструкции возникают повреждения покрытий и систем полов на миллионы долларов.Наборы для испытаний на выделение влаги определяют выделение влаги через бетонные плиты перекрытия с течением времени. Емкость с влагопоглощающим хлоридом кальция взвешивается и помещается под пластиковый купол, герметично прилегающий к бетонной поверхности с помощью самоклеющейся прокладки. После цикла испытаний пластик разрезают, и чашу с хлоридом кальция вынимают, герметизируют и взвешивают. Значения увеличения веса и времени воздействия используются для расчета результата теста, выраженного в фунтах влаги, выделяемой на 1000 квадратных футов за 24 часа.Измеритель влажности также может быть полезен для мгновенного измерения содержания влаги на бетонных поверхностях пола перед нанесением напольных покрытий.
    • Когда использовать Комплект или измеритель уровня влажности полезен при определении влажности глубоко под бетонной поверхностью. Этот тип теста на влажность пола также полезен при оказании помощи подрядчикам в выявлении подозрительных участков, которые могут потребовать дальнейшего тестирования, более глубоко в плите.
  • Системы измерения влажности бетона — Избыточная влажность бетонных полов может привести к дорогостоящему напольному покрытию или поломкам покрытия, таким как расслоение, коробление, образование пузырей и повышение вероятности роста плесени.Системы измерения относительной влажности (RH) предлагают полный профиль влажности по всей плите. Оператор просто пробуривает скважину на заданную глубину, а электронные датчики влажности периодически измеряют уровень влажности. После завершения испытания отверстия можно легко заполнить стандартным цементным раствором.
    • Когда использовать Система измерения относительной влажности полезна для измерения влажности и других факторов, включая температуру, точку росы и проверку влажности поверхности бетона в соответствии с ASTM F2659.
  • Локаторы арматуры и измерители покрытия — Локаторы арматуры и измерители покрытия используются для поиска арматурных стержней, сварной проволочной сетки и металлических стяжек в конструкциях. Их основная функция — установить вертикальное расположение стержней, чтобы избежать повреждения армирующих элементов во время резки или удаления керна. Расширенные модели позволяют оценить размер и глубину стержня для оценки целостности существующих конструкций или их соответствия проектным спецификациям.
    • Когда использовать Эти методы полезны для выявления точного размера, местоположения и глубины арматурной стали и подземных металлических конструкций для контроля качества и эффективного восстановления испытательных образцов.

    Мы надеемся, что наше руководство облегчило выбор идеального метода тестирования. Чтобы увидеть полный список, посетите нашу страницу оборудования для неразрушающего контроля!

    Глоссарий терминов, связанных с бетоном

    А

    Закон Абрамса — Правило бетона, устанавливающее, что при заданных материалах, условиях отверждения и испытаний,
    Прочность бетона обратно пропорциональна соотношению воды и цемента. Низкое соотношение воды и цемента обеспечивает более высокую прочность бетона.

    Ускоритель — Химическое вещество, добавляемое в бетонную смесь, сокращающее время схватывания за счет увеличения скорости гидратации.

    Заполнитель — смесь песка, камня, щебня, вспененных материалов или частиц, которые улучшают структурные характеристики.
    производительность бетона и улучшает образование и текучесть цементного теста. Заполнитель обычно составляет около 75% объема бетона.

    Воздухововлекающий- Бетон с воздухововлекающими добавками содержит мельчайшие пузырьки воздуха, которые равномерно распределены по всей цементной пасте.
    Воздух в бетоне может быть получен путем использования воздухововлекающего цемента, путем введения воздухововлекающей добавки,
    или комбинацией обоих методов.Воздухововлекающие добавки используются для стабилизации микроскопических пузырьков воздуха в бетоне.
    Правильный воздухововлечение с соответствующим объемом и коэффициентом зазора значительно повысит долговечность бетона.
    подвергается воздействию влаги во время циклов замораживания и оттаивания. Вовлеченный воздух также улучшает сопротивление бетона поверхности.
    накипь вызывает химические антиобледенители.

    Кислотное травление — Применение соляной или фосфорной кислоты для очистки или профилирования бетонной поверхности. Используется как альтернатива абразивно-струйной очистке для подготовки поверхности.

    Кислотное пятно — (или химическое пятно) Пятно, содержащее неорганические соли, растворенные в кислотном растворе на водной основе, который химически вступает в реакцию с минералами в затвердевшем бетоне с образованием стойкого прозрачного цвета, который не отслаивается и не отслаивается. Придает бетону привлекательный пестрый или мраморный вид. Цвета обычно бывают землистыми, такими как загар, коричневый, красновато-коричневый и зеленый.

    Acrylic Sealer- Акриловый растворитель или герметик на водной основе, который образует защитную пленку на поверхности бетона, придающую блеск, подчеркивающий красоту обычного и декоративного бетона.

    Добавка — Ингредиент в бетоне, кроме воды, портландцемента и заполнителя, используемый для изменения свойств бетона в его свежезамешенном, схватившемся или затвердевшем состоянии. Может добавляться в бетон на бетонном заводе или на стройплощадке. Доступны предварительно упакованные добавки для удобного добавления на строительную площадку, что дает подрядчикам возможность модифицировать бетон, который они получают, когда это необходимо, например, увеличивая количество времени, доступное для декоративной штамповки. См. Http: // www.geigerreadymix.com/

    Содержание воздуха — Количество увлеченного или захваченного воздуха в бетоне, обычно выражаемое в процентах от общего объема.

    ASR (Реакционная способность щелочи и кремнезема) — Реакционная способность щелочного кремния и кремния — это процесс, в котором определенные минералы (в основном кремнезем стеклянного типа) в присутствии влаги разрушаются в сильно щелочной среде бетона с образованием геля, который расширяется, растягивающие силы в бетонной матрице, вызывающие растрескивание бетона.Затем растрескивание позволяет большему количеству воды проникать в бетон, образуя больше геля, большее расширение и т. Д. В конечном итоге бетон разрушается или распадается.

    Antiquing- Техника нанесения цветных слоев для придания декоративным бетонным поверхностям состаренного или пятнистого вида.

    Американский институт бетона (ACI) — Международная организация, предоставляющая образование, сертификацию и информацию о бетоне.

    Американский национальный институт стандартов (ANSI) — Представитель США в Международной организации по стандартизации (ISO).

    Американский стандарт испытательных материалов (ASTM) — Организация, которая служит стандартизированным испытательным центром для оценки характеристик различных строительных материалов.

    Фартук — Бетонная плита, часто у въезда для транспортных средств, которая выходит за пределы входа в здание.

    B

    Отводная вода (стравливающая) — Вода, которая поднимается на поверхность свежеуложенного бетона из-за расслоения. Кровотечение может помешать завершающим операциям.Если на бетонную поверхность наносится отвердитель цвета сухого встряхивания, необходимо некоторое количество стравливаемой воды, чтобы отвердитель достаточно смачивался, чтобы его можно было вылить на поверхность.

    Вздутие — Образование пузырей на топпингах или покрытиях и потеря адгезии с нижележащим субстратом. На бетонных поверхностях это часто вызвано проблемами проникновения влаги или паров влаги.

    Связка — Степень адгезии или сцепления материала (например, покрытий, топингов, ремонтных растворов или герметиков) с существующей поверхностью.

    Связующий агент — Клейкий агент, используемый для увеличения сцепления покрытий или верхних слоев с существующей поверхностью. Также используется для приклеивания нового бетона к старому. Также известен как грунтовка.

    Разрыватель адгезии — Материал, предотвращающий прилипание материалов к бетонной основе.

    Broadcast — Наносить вручную отвердитель цвета, декоративный заполнитель или другой сухой материал в виде сухого встряхивания равномерным слоем на свежий бетон, перекрытия или покрытия для придания цвета или сцепления.(См. Также посев.)

    Отделка щеткой — Текстура поверхности, полученная при проталкивании щеткой свежеуложенного бетона.

    Поплавок для быка — Инструмент, состоящий из большого плоского прямоугольного лезвия, сделанного из дерева, смолы, алюминия или магния. Используется для устранения высоких и низких пятен на свежеуложенных бетонных плитах, заделки крупного заполнителя на поверхности, нанесения на поверхность слоя пасты, необходимого во время окончательной отделки, и нанесения пасты в отвердитель цвета сухого перемешивания. Длинные ручки либо прикрепляются, либо ввинчиваются в головку поплавка, чтобы его можно было вытолкнуть на плиту, пока пользователь стоит по периметру.

    С

    Хлорид кальция — Ускоряющая добавка для сокращения времени схватывания бетона в более прохладных влажных погодных условиях за счет увеличения скорости гидратации. Хлорид кальция может вызвать появление пятен на поверхности и вызвать коррозию стальной арматуры в бетонной плите.

    Заливка — Бетон уложен и закончен на своем окончательном месте.

    Цемент — Цемент — это не то же самое, что бетон, а, скорее, один из компонентов бетона.Цемент, комбинация мелко измельченных материалов, затвердевает при смешивании с водой и становится «клеем» в бетоне.

    Цементный — Материал, содержащий портландцемент в качестве одного из компонентов или обладающий свойствами, подобными цементу.

    Стулья (опоры) — Стулья с опорой для арматуры позволяют поддерживать стержни арматуры на устойчивом уровне по всей бетонной плите.

    Меление — Рыхлое порошкообразное вещество, образовавшееся в результате разрушения бетонной поверхности или разрушения покрытия или верхнего слоя.

    Прочность на сжатие — Максимальное напряжение сжатия, которое могут выдержать бетон или цементные перекрывающие материалы, выражается в фунтах на квадратный дюйм (psi).

    Бетон — Бетон — это не то же самое, что цемент. Бетон — это сочетание цемента в качестве связующего, химических добавок, воды и минерала. Эта комбинация при правильном смешивании и размещении превращается в отличный строительный материал для самых разных целей.

    Бетонные столешницы — Изготовленная вручную альтернатива искусственным поверхностям столешниц.Может быть изготовлен в магазине в формах, построенных по спецификациям клиентов, или отлит на месте, путем установки формы на базовые кухонные шкафы и последующего заполнения бетоном. Использование красителей, пигментов, декоративных заполнителей и эпоксидных покрытий может придать бетонным столешницам вид, текстуру и ощущение добытого камня, такого как мрамор, гранит и известняк.

    Консистенция — Способность свежего бетона течь. Обычная мера стабильности — спад.

    Контрольный (или усадочный) стык — Паз в бетонной плите, пропиленный или обработанный специальными инструментами, используется для регулирования места образования трещин.

    Строительный шов — — это поверхности, на которых встречаются две последовательные укладки бетона. Обычно их кладут в конце рабочего дня, но могут потребоваться, когда укладка бетона прекращается на время, превышающее начальное время схватывания бетона. В плитах они могут быть спроектированы так, чтобы допускать перемещение и / или передачу нагрузки.

    Вырезка трещин — Обработка трещин в бетоне пилой или угловой шлифовальной машиной перед заполнением ремонтным материалом.

    Трещины движущиеся — Трещины в бетоне, которые все еще движутся или действуют.Часто они имеют структурный характер и продолжаются на всю глубину бетона.

    Трещины, статические — Случайные неподвижные микротрещины, поражающие только бетонную поверхность (см. Также трещины трещин и трещины пластической усадки).

    Трещины — Серия мелких случайных трещин, вызванных усадкой строительного раствора.

    Покрытие — Состояние, которое возникает, когда поверхность свежеуложенного бетона высыхает слишком быстро, часто из-за воздействия прямых солнечных лучей, ветра, высоких температур или штампованного бетона.

    Кубический двор — Единица измерения товарной бетонной смеси. Бетон заказывается, продается и дозируется по объему.

    Отверждение — Действие, предпринимаемое для поддержания благоприятных условий влажности и температуры свежеуложенного бетона или цементных материалов в течение определенного периода времени после укладки. Помогает обеспечить достаточное увлажнение и правильное затвердевание.

    Отвердитель — Жидкость, которая при нанесении на поверхность только что уложенного бетона образует мембрану на бетоне или проникает в бетон, чтобы замедлить испарение воды.

    D

    Darby — Более длинная версия ручного поплавка длиной от 2 до 4 футов. Полезно для выравнивания проблемных зон.

    Декоративный заполнитель — Богато окрашенные природные камни, такие как базальт, гранит, кварц или известняк, используемые для улучшения внешнего вида бетона или декоративных покрытий.

    Декоративный бетон — Бетон, усиленный цветом, рисунком, текстурой или комбинацией декоративных обработок.

    Degreaser — Химический раствор для удаления жира, масел и других загрязнений с бетонных поверхностей.

    Отслаивание — Отделение покрытия или топинга от основы или слоев покрытия друг от друга из-за плохой адгезии. Или в случае бетонной плиты — горизонтальный раскол или отрыв верхней поверхности.

    Densifier — Проникающий жидкий химический отвердитель, наносимый на бетон, чтобы помочь укрепить и уплотнить поверхность и обеспечить дополнительную защиту от проникновения воды и образования пятен. Часто рекомендуется для полированного бетона, потому что твердый бетон обеспечивает лучшую полировку.

    Сухой отвердитель цвета — Смесь красящих пигментов, цемента, заполнителей и средств для кондиционирования поверхности. Применяется в виде сухого встряхивания для штампованного бетона или штампованных накладок для получения цветной износостойкой поверхности.

    Усадка при высыхании — Уменьшение объема бетона по мере высыхания из-за потери влаги. См. Также пластические усадочные трещины.

    Красители — Полупрозрачные цветовые решения, содержащие очень мелкие пигменты, проникающие в бетонную поверхность.Не вступает в химическую реакцию с бетоном (в отличие от кислотных пятен). Доступны как красители на водной основе, так и на основе растворителей, от мягких пастельных тонов до более смелых оттенков.

    E

    Edger — Инструмент, используемый для обработки кромок свежего бетона для получения чистой, обработанной кромки.

    Гравировка — Использование специальных инструментов и оборудования для вырезания или фрезерования узоров и рисунков в затвердевшем бетоне. Обычно бетон сначала окрашивают, чтобы придать ему цвет, поэтому фрезерованные участки выглядят как линии затирки.

    Выцветание — Кристаллический отложение солей (обычно белого цвета), которое образуется на поверхности бетона, когда растворимые гидроксиды кальция выщелачиваются из бетона и соединяются с диоксидом углерода в атмосфере. На цветном бетоне, особенно темных тонов, эти белые отложения могут быть особенно неприглядными.

    Эпоксидное впрыскивание — Метод для заделки или ремонта трещин в бетоне путем впрыскивания эпоксидного клея под низким давлением.

    Эпоксидные смолы — Органические химические связующие системы, используемые при приготовлении защитных и декоративных покрытий для бетона, клеев для инъекций в бетон с трещинами или в качестве связующих в эпоксидных растворах.

    Замедлитель испарения — Пленка на водной основе, наносимая распылением, которая временно снижает потерю влаги при нанесении на поверхность свежеуложенного бетона.

    Открытый заполнитель — Декоративная поверхность, образованная путем удаления поверхностного раствора с бетонной плиты (путем скребка, мойки под давлением или абразивоструйной очистки), чтобы обнажить нижележащие заполнители. (Также см. Замедлитель схватывания поверхности.)

    F

    Растушевка — Для плавного и плавного смешивания кромки покрывающего или ремонтного материала с существующим бетоном.

    Волокна — Крошечные волокна из полипропилена, полиолефина, нейлона, полиэтилена, полиэстера или акрила, используемые отдельно или в сочетании с арматурой или сварной проволочной сеткой для армирования бетона.

    Отделка — Выравнивание, разглаживание, уплотнение и иная обработка поверхности недавно уложенного бетона или бетонных покрытий для получения желаемого внешнего вида и эксплуатационных свойств.

    Пленкообразующий герметик — Тип герметика, который блокирует проникновение воды и загрязнений, образуя барьер на бетонной поверхности.Может также придавать блеск или блеск, которые усиливают окрашенный или незащищенный бетон. Также мембрана.

    Мигающий (или флэш-трансляция) — Техника нанесения акцентных цветов отвердителя методом сухого встряхивания на бетонные поверхности перед штамповкой. Приводит к появлению тонких, естественных цветовых вариаций.

    Прочность на изгиб — Способность затвердевшего бетона или верхнего слоя противостоять разрушению при изгибе.

    Финишная обработка — Текстура поверхности (обычно шероховатая), полученная финишной обработкой поплавком или ручной теркой.

    Полировальный станок — Мотоблок, используемый для производства полированного бетона. Большинство станков оснащено планетарной системой привода, большой головкой для первичной полировки (от 17 до 36 дюймов в диаметре), оснащенной тремя или четырьмя сателлитами меньшего размера, которые удерживают алмазный абразив. Во время работы машины вспомогательные головки вращаются в направлении, противоположном направлению первичной головки, чтобы устранить следы шлифования на полу. (См. Также алмазное шлифование, сухое полирование, влажное полирование).

    Летучая зола — Дополнительный вяжущий материал (SCM) Летучая зола является побочным продуктом, возникающим при сжигании измельченного или порошкообразного угля; иногда используется в качестве замены цемента в бетоне.

    Опалубка — Материал, используемый для облицовки внутренней поверхности опалубки с целью придания архитектурной отделки гладкой или узорчатой.

    Fresno — Большой шпатель (примерно от 2 до 4 футов в длину), используемый для окончательной отделки после спуска. Длинные ручки (например, те, которые используются для поплавков) либо пристегиваются, либо ввинчиваются в лезвие.ПРИМЕЧАНИЕ: Fresnos никогда не следует использовать для наружного бетона с воздухововлекающими добавками, поскольку он герметизирует верхнюю часть плиты, задерживая стекающую воду под поверхностью.

    Отделка — Обработка (уплотнение, выравнивание, выравнивание и т. Д.) Недавно уложенного или свежего бетона для обеспечения запланированной поверхности.

    Плоские конструкции — В основном бетонные работы с ровной поверхностью, включая тротуары, проезды, подвалы и бетонные полы.

    G

    Грабли — Инструмент с регулируемым ограничителем глубины, предназначенный для нанесения толстослойных покрытий или цементных покрытий с заданной равномерной толщиной.

    Гравийная смесь — Бетонная смесь, в которой в качестве крупного заполнителя используется мелкий гравий или более крупный гладкий гравий. Эта смесь типична для отделки из открытого заполнителя.

    Оценка — Строительный участок (или здание) существующий или предполагаемый уровень земли или возвышение. Также используется для категоризации зданий или уровней строительства, например выше, на уровне или ниже.

    Гранулированный доменный шлак — Дополнительный вяжущий материал (SCM), образующийся при быстром охлаждении расплавленного доменного шлака.Измельченный гранулированный шлак иногда используется в бетонных смесях в качестве замены цемента, чтобы помочь снизить проницаемость и повысить долговечность. Может также замедлить схватывание и продлить рабочее время бетона. Снижает вероятность щелочно-кремнеземной реакции.

    Шлифование — Метод механической подготовки поверхности с использованием вращающихся абразивных камней или дисков для удаления тонких покрытий и мастики или небольших трещин и выступов.

    Затирка — Смесь вяжущих материалов и воды, с заполнителем или без него, в пропорции для получения кремообразной консистенции.Можно приобрести предварительно смешанные во множестве цветов, чтобы определить швы и пропилы в декоративных бетонных плитах или стенах, особенно с камнем, кирпичом или плиткой.

    Groover — Инструмент с V-образной коронкой, используемый для создания контрольных швов в пластичном бетоне.

    H

    Ручной поплавок — Меньшая портативная версия поплавка для быков, длина от 12 до 18 дюймов. Особенно полезен для плавания по периметру форм или для работы в тесноте.

    Твердое шлифование — Обработка поверхности, полученная с помощью шпателя со стальным лезвием для окончательной отделки бетона. Часто используется там, где требуется гладкая, твердая, плоская поверхность.

    Водоструйная очистка под высоким давлением — Процесс очистки или придания шероховатости бетонным поверхностям с использованием струи воды, подаваемой под высоким давлением.

    Распылитель большого объема под низким давлением (HVLP) — Распылительное устройство, которое наносит краски и покрытия с высоким содержанием сухого остатка при низком давлении и низкой скорости, чтобы уменьшить избыточное распыление.

    Пистолет с бункером — Система с гравитационной подачей для нанесения покрытий или топпингов распылением. Материал помещается в бункер, прикрепленный к пистолету-распылителю, который приводится в действие воздушным компрессором. Часто используется для нанесения распылительных систем.

    Hover Trowel — Запатентованный легкий механический мастерок, разработанный специально для точной отделки эпоксидных, модифицированных полимером и цементных систем перекрытия.

    Гидратация — Химическая реакция между цементом и водой, вызывающая затвердевание бетона или других материалов на основе цемента.

    I

    Интегральный цвет — Краситель, предварительно смешанный с свежим бетоном или цементными покрытиями перед нанесением.

    Оксид железа — Неорганический пигмент, часто используемый для окрашивания декоративных покрытий и топингов.

    Изоляционный шов — Изоляционный (или компенсационный) шов отделяет или изолирует плиты от других частей конструкции, таких как стены, опоры или колонны; и проезды и патио от тротуаров, гаражных плит, лестниц, фонарных столбов и других точек ограничения.Они допускают независимое вертикальное и горизонтальное перемещение между прилегающими частями конструкции и помогают минимизировать растрескивание, когда такие движения сдерживаются.

    Дж

    Соединение (контроль, расширение или изоляция) — Формованная, пропиленная или обработанная канавка в бетонной плите, используемая для регулирования местоположения трещин (контрольный стык) или для обеспечения возможности расширения или перемещения прилегающих конструкций. В декоративном бетоне стыки также могут служить очерчивающими элементами дизайна в узоре.См .: http://www.geigerreadymix.com/portals/0/CIP_Joints_In_Slabs_On_Grade.pdf

    Заполнитель швов — Сжимаемый материал, используемый для заполнения шва для предотвращения проникновения мусора.

    К

    Пропил — Разрез в бетонной поверхности, сделанный пилой или фрезером.

    Смесь KCMMB — Спецификация бетонной смеси, разработанная Советом по материалам метро Канзас-Сити для бетона с более длительным сроком службы, путем создания требований к конструкции смеси из конкретных материалов и заполнителей твердых пород, которые могут смягчить воздействие солей, антиобледенителей и щелочно-кремнеземистых веществ. реактивность.

    Подножки — Подножки, используемые специалистами по отделке бетона для того, чтобы стоять на коленях при ручном затирке или затирке бетонных поверхностей.

    Постепенное покрытие — Достигается путем нанесения декоративного покрытия с помощью пистолета-распылителя и последующего сбивания материала шпателем для получения гладкой или слегка текстурированной поверхности.

    л

    Молоко — Тонкий слой мелких, слабо связанных частиц на поверхности свежего бетона, образованный восходящим движением воды.Цементное молоко необходимо удалить перед нанесением декоративного покрытия или топпинга.

    M

    Marbleize — Придает бетонным поверхностям вид и блеск мрамора за счет сочетания цветовых слоев и методов отделки.

    Мастерок по краям — Стальной шпатель с маленьким прямоугольным плоским лезвием длиной от 5 до 8 дюймов и короткой ручкой. Он имеет множество применений, включая соскабливание бетона с инструментов для отделки и нанесение ямочных материалов.

    Маскирование — Покрытие отдельных участков бетонной поверхности клейким трафаретом, лентой или другим материалом перед нанесением декоративной обработки, которая затронет только открытые участки.

    Паспорт безопасности материала (MSDS) — Информационные листы, содержащие соответствующие химические ингредиенты, правила обращения с продуктом и правила безопасности.

    Мембрана — Формируется на бетонной поверхности для защиты и улучшения цвета. Обычно прозрачный пластик, такой как акрил, полиуретан или эпоксидная смола.

    Microtopping — Ультратонкий декоративный слой на полимерной основе, как правило, общей толщиной менее 1/4 дюйма. Обычно наносится шпателем или ракелем и придает текстуру или гладкость.Пигменты могут быть включены в смесь или нанесены на поверхность для придания мраморности.

    Mil — Размер, равный 1/1000 (0,001) дюйма. Обычно используется для обозначения толщины покрытия.

    Состав смеси — Конкретные пропорции ингредиентов (цемент, заполнители, вода и добавки), используемые для производства бетона, подходящего для определенного набора рабочих условий.

    Mockup — Архитектурный образец бетона, изготовленный с использованием тех же материалов и тех же методов, что и для реального проекта.Часто требуется для обеспечения качества крупных проектов, чтобы гарантировать соблюдение архитектурных требований и отраслевых допусков. Размер должен быть достаточным, чтобы адекватно продемонстрировать все декоративные обработки.

    Пропускание влаги и паров — Миграция паров влаги к поверхности бетонной плиты, вызванная перепадами давления паров в бетоне и окружающей атмосфере. Может способствовать разрушению непроницаемых покрытий или других покрытий пола, не пропускающих влагу.

    N

    Neutralize — Для восстановления правильного pH бетона после кислотного травления, обычно путем промывания поверхности смесью воды и аммиака или карбоната натрия. Идеальный pH составляет 7,0 (нейтральный), но диапазон pH 6,0-9,0 приемлем для большинства покрытий. ASTM D 4262, «Стандартный метод испытания pH химически очищенных или протравленных бетонных поверхностей», описывает процедуру определения кислотности или щелочности бетонных поверхностей, подготовленных химической очисткой или травлением перед нанесением покрытия.

    Ракель с зазубринами — Резиновый ракель с насечками или зубцами на одной или обеих кромках. Используется для равномерного и равномерного нанесения продуктов из эпоксидной смолы или других покрытий с низкой вязкостью.

    O

    Непрозрачность — Способность покрытия скрывать цвет подстилающей поверхности. См. Полупрозрачный.

    Накладка — Связанный слой материала толщиной от 1/4 до 1 дюйма или более, размещаемый на существующих бетонных поверхностях для украшения, выравнивания или восстановления.

    -п.

    Бетон с рисунком — См. Штампованный бетон

    Проникающий герметик — Герметик со способностью проникать в бетонную поверхность для повышения водоотталкивающих свойств. Проникающие герметики вступают в химическую реакцию с бетоном, предотвращая проникновение воды и соли сквозь бетон.

    Проницаемость — Степень, в которой мембрана или покрытие допускают прохождение или проникновение жидкости или газа.

    Проницаемый бетон — Проницаемый бетон — это особый тип бетона с высокой пористостью, используемый для бетонных плоских работ, который позволяет воде от атмосферных осадков и других источников проходить через него, тем самым уменьшая сток с участка и подпитывая уровни грунтовых вод.

    PH Test — Тест, проводимый на бетонной поверхности для определения уровня кислотности или щелочности. Обычно выполняется перед нанесением герметиков или покрытий.

    Пигмент — Тонко измельченная натуральная или синтетическая частица, придающая цвет и непрозрачность покрытию или топингу.

    Размещение — Физическое введение бетонной смеси в конечное место, где она должна затвердеть и затвердеть.

    Пластик — Состояние свежезамешенного бетона, указывающее на то, что он поддается обработке и легко формуется.

    Трещины из-за пластической усадки — Неправильные трещины, которые возникают на поверхности свежего бетона вскоре после его укладки и пока он еще пластичный.

    Пластификатор — Водоредуцирующая добавка, позволяющая увеличить осадку без увеличения водоцементного отношения.

    Polar Set — Нехлоридная (неагрессивная) ускоряющая добавка для уменьшения времени схватывания бетона в более прохладных и влажных условиях за счет увеличения скорости гидратации.

    Полированный бетон — Глянцевое покрытие, достигаемое за счет использования специальных полировальных машин, оснащенных абразивными дисками с алмазной пропиткой (аналогично наждачной бумаге) для шлифовки поверхностей до желаемой степени блеска и гладкости. Получающаяся в результате поверхность не требует особого ухода и может быть окрашена, чтобы имитировать внешний вид полированного камня.

    Покрытие, модифицированное полимером — Покрытие на основе цемента с добавлением полимерных смол для улучшения рабочих характеристик, износостойкости и эстетических качеств.Производители оверлейных покрытий используют различные типы полимерных смол, часто смешивая их для производства запатентованных продуктов с уникальными характеристиками. Во многих современных декоративных накладках используются смеси акрила или винила, потому что эти смолы обеспечивают отличную прочность сцепления и стойкость к ультрафиолетовому излучению.

    Полимерная морилка — Морилка на акрилово-уретановой основе, доступная в более широкой цветовой палитре, чем кислотные морилки. Очень низкое содержание летучих органических соединений, удобоукладываемость аналогична латексной краске.Может наноситься на бетонные поверхности кистью, валиком, губкой, тканью или обычным распылителем.

    Выскакивающий — Яма или кратер на поверхности бетона размером от 1/4 дюйма до нескольких дюймов в диаметре, возникающий в результате разрушения несостоятельных частиц заполнителя из-за давления расширения. Обычно вызвано пористым заполнителем, имеющим высокую скорость абсорбции.

    Портландцемент — Гидравлический продукт, который схватывается и затвердевает при химическом взаимодействии с водой.Изготовлен путем обжига смеси известняка и глины или подобных материалов.

    Пуццолан — Кремнистый и глиноземистый материал, который в присутствии влаги химически реагирует с гидроксидом кальция с образованием соединений, обладающих вяжущими свойствами.

    Грунтовка — Первый слой материала, наносимый на бетонную поверхность для улучшения сцепления или адгезии последующих слоев. См. Также связующее покрытие.

    Накачивающий распылитель — Безвоздушный распылитель, часто используемый для нанесения герметиков и жидких разделительных агентов.

    R

    Raveling — Смещение заполнителя по краям швов или зазубрин в бетоне, как правило, вызванное распиловкой швов слишком скоро после укладки бетона.

    Товарный бетон — Бетон, который дозируется или смешивается на центральном заводе перед доставкой на строительную площадку для укладки.

    Арматура (или арматурные стержни) — Ребристые стальные стержни, устанавливаемые в монолитный бетон для обеспечения прочности на изгиб. Арматура бывает разных диаметров и классов прочности.

    Входящий угол — Угол в бетонной плите, обращенный внутрь. Часто подвержен растрескиванию, если не установлен управляющий шарнир.

    Железобетон — Бетонная конструкция, в которую встроена стальная арматура или сварная проволочная сетка для обеспечения большей устойчивости к растяжению и изгибным напряжениям.

    Растрескивание при отражении — Возникновение трещин в покрытиях и покрытиях, которые совпадают с местами существующих трещин в основании.

    Разделительный агент — Порошок или жидкий разделительный агент, наносимый на маты для штамповки или текстурированные покрытия перед штамповкой для предотвращения прилипания матов к свежему покрытию или бетонным поверхностям.

    S

    Мешок — Некоторое количество портландцемента. Один мешок весит 94 фунта.

    Смесь для мешков — Количество мешков с цементом на кубический ярд бетона. При заказе указывается, что бетон обычно называется смесью из 5 мешков (или мешков 5,5, мешков 6 и т. Д.).Мешки с цементом, необходимые для смеси, обычно указываются либо в планах, либо в спецификациях проекта. Больше мешков = больше прочности.

    Соляная отделка — Текстурированная декоративная отделка, получаемая путем рассыпания каменной соли на свежий бетон, а затем с помощью валика или терки для вдавливания частиц соли в поверхность. После схватывания бетона соль смывается, и на нем появляется пятнистый узор из неглубоких углублений.

    Пескоструйная обработка — Метод шлифования или профилирования поверхности струей песка, выбрасываемой из сопла с высокой скоростью сжатым воздухом.

    Пескоструйная обработка трафаретов — Техника создания рисунка на существующих бетонных поверхностях путем нанесения эластичных клеящих трафаретов с последующей пескоструйной очисткой для легкого удаления бетона только на открытых участках.

    Распиловка — Использование пилы по бетону с абразивными лезвиями или дисками для прорезания стыков или надрезания рисунков в затвердевшем бетоне.

    Накипь — Отслаивание или отслаивание затвердевшей бетонной поверхности, часто из-за воздействия замерзания или оттаивания.В отличие от отслаивания, отслаивания или скалывания бетона.

    Стяжка — Операция формирования поверхности с помощью стяжки или направляющих для стяжки и стяжки.

    Герметик — Материал на основе растворителей или жидкости, используемый для защиты и улучшения внешнего вида декоративного бетона.

    Посев — Распространение декоративных заполнителей на поверхность свежеуложенного бетона или покрытий.

    Разделение — Разделение компонентов влажного бетона, вызванное чрезмерным обращением или вибрацией.

    Самовыравнивающийся слой — Текучий полимерцементный цементный слой с возможностью самовыравнивания без затирки. Используется для сглаживания и выравнивания существующих бетонных поверхностей. Также может быть усилен окрашиванием, окрашиванием или распиловкой.

    Set — Состояние, достигаемое бетоном при потере пластичности, обычно измеряемое с точки зрения сопротивления проникновению или деформации. Первоначальное схватывание относится к бетону, достигшему первой степени застывания. Окончательное схватывание происходит, когда бетон достигает полной жесткости.

    Установка — Химическая реакция, которая происходит после добавления воды в цементную смесь, приводящая к постепенному развитию жесткости.

    Замедлители схватывания — химическая добавка, которая замедляет время схватывания бетона. Используется в жарких условиях для предотвращения слишком быстрого схватывания бетонной смеси для надлежащего набора прочности.

    Дробеструйная очистка — Метод абразивно-струйной обработки с использованием круглой дроби для очистки и профилирования бетонных поверхностей.

    Погрузчик с бортовым поворотом

    Нижний слой — Верхний слой, наносимый очень тонким слоем с помощью ракеля или шпателя.

    Шлак — см. Шлак доменный гранулированный.

    Растворитель — Жидкость, обычно используемая в качестве носителя для герметиков и отвердителей.

    Отслаивание — Отслаивание бетона в стыках полов или плит. Обычно возникает на соединениях, которые установлены неправильно или не выдерживают приложенные к ним нагрузки.

    Выкрашивание — вызывается попаданием воды в бетон и заставляет поверхность отслаиваться, выскакивать или отслаиваться. Выкрашивание бетона наиболее вероятно на внешних поверхностях, подверженных циклам замерзания и оттаивания. Причины включают: высокое водоцементное соотношение в бетонной смеси, отсутствие воздухововлечения, неправильная отделка и недостаточное отверждение.

    Система распыления — Декоративная накладка, наносимая в качестве слоя разбрызгивания или отделки методом «нокдауна» до толщины около 1/8 дюйма.Часто используется вместе с бумажными или клеящимися трафаретами. Доступны предварительно окрашенные или могут быть полностью окрашены во время смешивания.

    Осадка — Мера консистенции свежезамешенного бетона, определяемая расстоянием, на которое бетон оседает после того, как формованный образец вынут из перевернутого воронкообразного конуса.

    Штампованный бетон — Плоские бетонные конструкции с рисунком платформенных инструментов, штамповочных матов или бесшовных текстурных покрытий, напоминающих такие материалы, как кирпич, шифер, камень, плитка и деревянные настилы.

    Штампованное покрытие — Подобно обычному штампованному бетону, но может применяться к существующему бетону. Цементный слой наносится толщиной от 1/4 до 3/4 дюйма, а затем штампуется для имитации кирпича, сланца и натурального камня. Варианты цвета включают в себя отвердители красителей, окрашенные жидкими или порошкообразными разделителями, кислотные пятна, красители и тонированные герметики.

    Коврики для штамповки — Жесткие или полугибкие полиуретановые инструменты для нанесения рисунка на камень, шифер, кирпич и другие узоры на штампованных бетонных поверхностях.Коврики для штамповки обычно оставляют более мелкий рисунок, чем инструменты платформы.

    Бетон по трафарету — Декоративная обработка поверхности с использованием высокопрочных бумажных трафаретов с узорами из камня, плитки или кирпича, которые слегка вдавливаются в свежий бетон, с последующим нанесением отвердителей цвета сухого встряхивания. Когда трафареты удалены, неокрашенный бетон имитирует швы раствора. Другой метод, который можно использовать на существующем бетоне, заключается в нанесении клейких трафаретов, а затем окрашивании, травлении или пескоструйной очистке поверхности.(Также см. Трафарет пескоструйной обработкой.)

    Прямоугольник — Жесткий прямой кусок дерева или металла, используемый для отталкивания бетонной поверхности до нужного уровня перед плаванием.

    Strike off — Для выравнивания свежеуложенного бетона до нужной отметки.

    Основание — Существующая бетонная поверхность, на которую наносится верхнее покрытие, декоративное или защитное покрытие, проводится ремонт или другая шлифовка.

    Суперпластификаторы — Водоредуктор с широким диапазоном действия, который снижает содержание воды на 12–13 процентов и может добавляться в бетон с низкой или нормальной осадкой и отношением воды к цементу для получения бетона с высокой текучестью осадка.

    Замедлитель схватывания поверхности — Химическое вещество, наносимое на поверхность только что уложенного бетона для замедления схватывания цементного теста, чтобы его можно было легко удалить позже с помощью скребка или механической мойки для получения незащищенной отделки заполнителя.

    Т

    Тампер (или пундер) — Ручной ударный инструмент, используемый для плотного вдавливания штамповочных матов или текстурирующих покрытий в свежий бетон для обеспечения полного отпечатка.

    Технический паспорт — Содержит важные спецификации и инструкции производителя по использованию продукта.Включает такие данные, как степень покрытия, рекомендуемые области применения, ограничения продукта, рекомендации по подготовке поверхности, соотношения смешивания и требуемое время смешивания, жизнеспособность, процедуры нанесения, время отверждения, данные о характеристиках и меры предосторожности.

    Текстурный валик — Цилиндрический инструмент, похожий по внешнему виду на малярный валик, используемый для придания каменной текстуры бетону, нанесенному по трафарету. Его накладывают на трафарет и свежий бетон, чтобы текстурировать только открытые поверхности.

    Текстурированные оболочки — Гибкие оболочки для добавления бесшовных текстур на бетонные поверхности, не оставляя глубоких линий рисунка.Обычно более тонкие и податливые, чем коврики для штамповки. Часто используется для текстурирования периметра перекрытий и вертикальных граней, таких как подступенки лестниц. Также может использоваться для исправления дефектов от неравномерного тиснения.

    Оттенок — Разбавленная краска, используемая для придания оттенка декоративному бетону.

    Мастерок — Плоский ручной стальной инструмент с широким лезвием, используемый для уплотнения слоя пасты на поверхности и получения гладкой, ровной поверхности. Также полезно для нанесения покрытий или ремонтных материалов.Доступны разные формы (с закругленными или квадратными краями) и длины (от 8 до 24 дюймов). Маленькие шпатели подходят для бордюров, работы на ограниченных участках или для работы с высвечивающими акцентами отвердителя цвета сухого взбалтывания.

    Отделка шпателем — Гладкая или слегка текстурированная поверхность, полученная затиркой.

    В

    Пароизоляция — Влагонепроницаемый материал, такой как пластиковая пленка, помещается на основание под бетонной плитой для предотвращения проникновения паров влаги.

    Испытание на выброс пара — Испытание ASTM, используемое для измерения объема паров влаги, выделяемого из бетонной основы с течением времени (обычно 24 часа). Слишком много влаги, выделяемой из плиты, может повлиять на характеристики и склеивание перекрытий, покрытий и герметиков. Доступны наборы для проверки паров влаги, которые включают небольшие емкости с предварительно взвешенным негидратированным хлоридом кальция.

    Вертикальный штампованный бетон — Декоративная отделка стен и других вертикальных поверхностей с использованием легкого цементного покрытия, предназначенного для нанесения на толщину до 3 дюймов без провисания.Хотя покрытие все еще пластиковое, его можно штамповать или вырезать вручную для создания рельефных текстур камня или кирпичной стены. После высыхания материала кислотные пятна или красители можно распылить или нанести на поверхность губкой, чтобы получить ее в многокомпонентном смесителе.

    Вт

    Водоцементное соотношение — Отношение количества воды к количеству цемента в бетонной смеси. Ключом к производству высококачественного бетона является поддержание как можно более низкого водоцементного отношения без ущерба для удобоукладываемости.

    Водоредуктор — Добавка, которая либо увеличивает осадку свежезамещенного бетона без увеличения содержания воды, либо поддерживает удобоукладываемость с меньшим количеством воды, не влияя на прочность.

    Сварная проволочная сетка — Плетеная сетка из проволочных прядей, сваренных на каждом пересечении, используется для армирования бетонных плит. Также называется сварной проволочной сеткой.

    Мокрая полировка — Метод полировки бетона, при котором вода используется для охлаждения алмазных абразивов и удаления шлифовальной пыли.Не так часто, как сухая полировка, потому что в процессе образуется огромное количество суспензии (жидкая смесь воды и цементной пыли), которую необходимо собрать и утилизировать.

    Белый цемент — Портландцемент с низким содержанием железа, который гидратируется до белой пасты. Часто используется в цветном бетоне для получения чистых ярких тонов, особенно пастельных.

    Технологичность — Легкость, с которой бетон или другие вяжущие материалы можно смешивать, укладывать и отделывать.

    Рабочее время — Время, необходимое для укладки и отделки материала на цементной основе до того, как он начнет схватываться. Часто зависит от температуры окружающей среды и температуры основания.

    Х

    Ксилол — Обычный растворитель. Используется как носитель для герметиков на основе растворителей. С сильным запахом и воспламеняемостью.

    Виды добавок для бетона

    Добавки для бетона используются для улучшения поведения бетона в различных условиях и бывают двух основных типов: химические и минеральные.

    ХИМИЧЕСКИЕ ДОБАВКИ

    Корпорация Fritz-Pak в Далласе, Техас

    Химические добавки снижают стоимость строительства, изменяют свойства затвердевшего бетона, обеспечивают качество бетона во время смешивания / транспортировки / укладки / выдержки, а также позволяют избежать некоторых аварийных ситуаций во время бетонных работ.

    Химические добавки используются для улучшения качества бетона при смешивании, транспортировке, укладке и выдержке. Они делятся на следующие категории:

    • воздухововлекающие
    • редукторы воды
    • комплект замедлителей
    • набор ускорителей
    • суперпластификаторы
    • специальных добавок: в их состав входят ингибиторы коррозии, средства контроля усадки, ингибиторы щелочно-кремнеземной реакции и красители.

    Найти производителей: Магазин добавок

    МИНЕРАЛЬНЫЕ ДОБАВКИ

    Минеральные добавки делают смеси более экономичными, снижают проницаемость, повышают прочность и влияют на другие свойства бетона.

    Минеральные добавки влияют на характер затвердевшего бетона за счет гидравлической или пуццолановой активности. Пуццоланы являются вяжущими материалами и включают природные пуццоланы (такие как вулканический пепел, используемый в римском бетоне), летучую золу и пары кремнезема.

    Их можно использовать с портландцементом или цементом с добавками по отдельности или в комбинации.

    Категории ASTM — Добавки в бетон

    ASTM C494 определяет требования для семи типов химических добавок. Их:

    • Тип A: Водоредуцирующие добавки
    • Тип B: Замедляющие добавки
    • Тип C: Ускоряющие добавки
    • Тип D: водоредуцирующие и замедляющие добавки
    • Тип E: Водоредуцирующие и ускоряющие добавки
    • Тип F: водоредуцирующие добавки высокого диапазона
    • Тип G: водоредуцирующие, высокодисперсные и замедляющие добавки

    Примечание. Изменения в индустрии добавок происходят быстрее, чем согласованный процесс ASTM.Добавки, уменьшающие усадку (SRA) и средние водоредукторы (MRWD) — это две области, для которых в настоящее время не существует спецификаций ASTM C494-98.

    Рекомендуемые товары

    КАКИЕ ОСОБЕННОСТИ ВАМ НУЖНЫ?

    Что такое отслоенный бетон
    Время: 06:08
    Посмотрите это простое для понимания объяснение причин отслаивания бетона от эксперта по бетону Криса Салливана.

    Воздухововлечение

    Редукция воды

    Высокопрочный бетон

    Защита от коррозии

    Установить ускорение

    Установить замедление

    Текучесть

    Улучшители отделки

    Заливка жидкостью — (CLSM)

    Корпорация Fritz-Pak в Далласе, Техас

    Защита от замораживания Циклы оттаивания Повышение прочности

    Воздухововлечение особенно эффективно для обеспечения устойчивости к циклам замораживания-оттаивания.Когда влага в бетоне замерзает, эти воздушные ячейки сбрасывают внутреннее давление, создавая микроскопические камеры для расширения воды при замерзании.

    Некоторые воздухововлекающие смеси содержат катализатор для более быстрой и полной гидратации портландцемента.

    Для защиты бетона от повреждений при замерзании пузырьки должны иметь правильный размер, распределение и объем. ASTM C 260 устанавливает требования к воздухововлекающим добавкам.

    Корпорация Fritz-Pak в Далласе, Техас

    Преимущества воздухововлечения включают:

    • Повышенная устойчивость бетона к сильному морозу или циклам замораживания / оттаивания
    • Высокая устойчивость к циклам смачивания и высыхания
    • Высокая технологичность
    • Высокая прочность

    Дозировка: Обычно воздухововлечение составляет от 5% до 8% от объема бетона.

    Уменьшение количества воды в смеси

    Регуляторы воды стали настолько важными в бетоне, что их можно считать «пятым» ингредиентом.

    Их можно использовать для: (1) увеличения осадки, (2) снижения водоцементного отношения или (3) уменьшения содержания цемента.

    Редукторы воды бывают суперпластификаторами низкого, среднего и высокого диапазона. Доступно достаточно различных добавок, чтобы можно было выбрать ту, которая соответствует потребностям конкретного проекта, будь то высокие колонны, которым требуется легко перекачиваемая смесь, или легкая в отделке прочная плита перекрытия.

    Как правило, они обеспечивают требуемую осадку с меньшим количеством воды в смеси и могут обеспечить бетон более высокой прочности без увеличения количества цемента.

    Обычные редукторы воды

    Требуются для достижения минимального снижения содержания воды на 5%. Обычный редуктор воды может уменьшить оседание примерно на 1-2 дюйма без добавления воды.

    Редукторы среднего уровня

    Может снизить содержание воды от 8% до 15%.Они, как правило, стабильны в более широком диапазоне температур и дают более стабильное время схватывания. Эти редукторы наиболее эффективно работают в смесях, рассчитанных на оседание в диапазоне от 4 до 5 дюймов.

    • Особенно полезен при бетонировании в жаркую погоду, поскольку противодействует ускоряющему воздействию высоких температур окружающей среды и бетона

    • Замедление схватывания бетона

    • Отсрочка начального набора бетона

    • Сохраняйте работоспособность бетона во время укладки

    Высококачественные восстановители воды (суперпластификаторы)

    Может снизить содержание воды с 12% до 40% и обычно используется в бетонах, рассчитанных на просадку от 8 до 11 дюймов.Их можно использовать либо для увеличения осадки (на 4-8 дюймов), либо для снижения содержания воды в бетонных смесях для жаркой погоды. Используется для увеличения текучести. Вы также можете прочитать часто задаваемые вопросы о суперпластификаторах на сайте Fritz-Pak.

    Высокопрочный бетон

    Добавки, содержащие микродиоксид кремния (конденсированный дымок кремнезема), используются для удовлетворения требований высокой прочности и низкой проницаемости.

    Преимущества

    включают пониженную проницаемость, повышенную прочность на сжатие и изгиб, а также повышенную долговечность.

    Применяется в высокопрочных конструкционных колоннах, менее проницаемых настилах гаражей и стойких к истиранию гидротехнических сооружениях.

    Дым кремнезема можно использовать в бетоне для получения прочности на сжатие, приближающейся к 20 000 фунтов на квадратный дюйм в условиях стройплощадки. Эта добавка может быть добавлена ​​в виде суспензии или в сухой форме, в зависимости от того, что отвечает требованиям дозирующего оборудования. В любом случае производительность одинакова.

    Повышение прочности

    Повышение прочности бетона может быть достигнуто за счет использования добавок суперпластификатора для получения низкого водоцементного отношения, дающего бетон с высокими эксплуатационными характеристиками.

    Корпорация Fritz-Pak в Далласе, Техас
    Корпорация Fritz-Pak в Далласе, Техас

    Эти добавки способствуют образованию высокопрочного, чрезвычайно текучего бетона, который обеспечивает высокую прочность, обеспечивая при этом превосходную обрабатываемость и прокачиваемость.

    Высококачественные водоредуцирующие добавки также могут использоваться для сборных / предварительно напряженных конструкций, где желательно поддерживать соотношение вода / цемент на минимальном уровне для низкой проницаемости и высокой начальной прочности без замедления схватывания. Они также используются для бетона, требующего высокой прочности. сильные стороны раннего зачистки.

    Зола-унос — делает бетон более прочным, долговечным и легким в эксплуатации с

    Зола-унос, получаемая при сжигании угля, является ценной добавкой, которая делает бетон более прочным, долговечным и легким в эксплуатации.

    Летучая зола способствует образованию вяжущих смесей, повышающих прочность, непроницаемость и долговечность бетона.

    В бетоне используется два основных класса летучей золы: класс F и класс C.

    Класс F

    Уменьшает просачивание и расслоение пластичного бетона.В затвердевшем бетоне увеличивает предел прочности, уменьшает усадку при высыхании и проницаемость, снижает теплоту гидратации и снижает ползучесть.

    Класс C

    Обладает уникальными характеристиками самоотверждения и улучшает проницаемость. Особенно полезно в предварительно напряженном бетоне и других областях, где требуется высокая начальная прочность. Также полезен для стабилизации грунта.

    Дым кремнезема: ранняя прочность и пониженная проницаемость

    Дым кремнезема может внести значительный вклад в повышение прочности бетона в раннем возрасте.Один фунт микрокремнезема выделяет примерно такое же количество тепла, как фунт портландцемента, и дает примерно в три-пять раз большую прочность на сжатие.

    Дым кремнезема улучшает бетон двумя способами: основной пуццолановой реакцией и эффектом микронаполнителя. Добавление микрокремнезема улучшает сцепление с бетоном и помогает снизить проницаемость, а также в сочетании с гидроксидом кальция, образующимся при гидратации портландцемента, улучшает долговечность бетона.

    В качестве микронаполнителя чрезвычайно тонкая пыль кремнезема позволяет ему заполнять микроскопические пустоты между частицами цемента.Это значительно снижает проницаемость и улучшает сцепление полученного бетона с заполнителем по сравнению с обычным бетоном.

    Для получения дополнительной информации о микрокремнеземе и его преимуществах посетите:

    Дозировка:

    от 8% до 15% по массе цемента, но в качестве дополнения не заменяет
    от 8% до 10% Высокая прочность / низкая проницаемость, например, настилы мостов или парковочные конструкции
    от 10% до 15% Высокопрочные конструкционные колонны
    10% макс. Квартира

    Требуемое количество зависит от дозировки микрокремнезема и соотношения водоцементных материалов.Пары кремнезема являются вяжущими, но обычно их добавляют к и не заменяют существующий портландцемент.

    Совет: чем выше процент используемого микрокремнезема, тем больше требуется суперпластификатора, но смесь может стать «липкой». Рассмотрите возможность замены примерно 1/3 суперпластификатора средним водоредуктором для улучшения удобоукладываемости

    Использует:

    • Снижает проницаемость бетона
    • Повышает прочность бетона
    • Повышает устойчивость к коррозии

    Добавки для бетона, контролирующие растрескивание, уменьшающие высыхание или усадочные трещины

    Гидратированная цементная паста дает усадку, поскольку теряет влагу из своих чрезвычайно мелких пор.Поскольку влага теряется в этих маленьких порах, поверхностное натяжение оставшейся воды имеет тенденцию стягивать поры вместе, что со временем приводит к потере объема.

    Добавки, уменьшающие усадку (SRA), предназначены для уменьшения эффекта усадки при высыхании за счет уменьшения поверхностного натяжения в этих порах.

    Следует отметить, что тип заполнителя и сами свойства цемента могут влиять на количество возможных трещин. Таким образом, при испытании на усадку важно протестировать местные материалы для конкретного проекта.

    Отверждение также влияет на растрескивание. В плитах верх обычно высыхает первым и сжимается, в то время как нижние части все еще имеют более высокое содержание влаги. Эту разницу во влажности можно изменить с помощью добавок, уменьшающих усадку, которые изменяют способ миграции воды через бетон и приводят к более однородному профилю влажности.

    Защита от коррозии

    Железобетон, который подвергается воздействию солей для борьбы с обледенением и морской среде, особенно подвержен коррозии, вызванной хлоридом.

    Хорошо продуманная, прочная бетонная смесь с низкой проницаемостью обеспечит некоторую защиту арматуры от коррозии, вызванной хлоридом.

    Ингибиторы коррозии могут быть эффективны на стоянках, мостах и ​​в морской среде. Другими способами уменьшения коррозии являются использование проникающих поверхностных герметиков для предотвращения проникновения хлоридов, арматуры с эпоксидным покрытием или дыма кремнезема в смеси. Пары кремнезема могут увеличить долговечность и снизить проницаемость.

    Доступны нехлоридные добавки ускорителей схватывания, соответствующие стандарту ASTM C 494 Type C.

    Хлорид кальция использовался в прошлом, потому что он относительно недорог и обеспечивает ускорение схватывания и раннее развитие прочности. Однако коррозионные эффекты хлорида наблюдались спустя 20 лет после заливки бетона. Таким образом, нехлоридсодержащие добавки были разработаны для использования там, где необходимо избегать потенциальной коррозии заделанной или подвергнутой напряжению стали.

    Как и в случае со всеми другими добавками, для обеспечения эффективности необходимо соблюдать инструкции производителя по их использованию в сочетании с другими добавками и дозировками.

    Установить ускорение

    Корпорация Fritz-Pak в Далласе, Техас

    Ускорители схватывания работают за счет ускорения гидратации цемента, что приводит к сокращению времени схватывания и увеличению прочности в раннем возрасте, особенно при более низких температурах.

    Они увеличивают скорость раннего развития прочности и сокращают время, необходимое для отверждения и защиты.

    Когда-то хлорид кальция был преобладающей добавкой-ускорителем. Однако в настоящее время многие считают, что это является основным источником долгосрочных конкретных проблем.В Европе использование хлорида кальция для некоторых целей запрещено. Считается, что хлорид способствует коррозии арматуры или закладного металла в бетоне. В свою очередь, эта коррозия была связана с отслаиванием, растрескиванием, потерей сцепления и, если ее не исправить, может привести к возможному выходу из строя соответствующего элемента.

    Теперь доступны не содержащие хлоридов ускорители схватывания на основе других химикатов. Некоторые из этих новых добавок также могут действовать как водоредукторы ASTM C494 типа E.

    Задержка набора

    Корпорация Fritz-Pak в Далласе, Техас

    Замедлители схватывания используются там, где требуется задержка времени схватывания для обеспечения достаточного времени укладки, вибрации или уплотнения.

    Замедлители схватывания позволяют применять более высокотемпературное отверждение сборного / предварительно напряженного бетона без отрицательного влияния на предел прочности.

    Заявки:

    • Дальние перевозки
    • Грузовики ждут долго — даже за маленькую заливку
    • Неквалифицированная расстановочная бригада
    • Низкий расход
    • Штамповка бетона в теплую погоду

    На что следует обратить внимание:

    Местоположение проекта — дальние или ближние расстояния

    • Размер заливки
    • Скорость заливки
    • Способ размещения

    Окружающая среда снаружи, внутри

    Толстые секции — (в сухих ветреных условиях раннее высыхание поверхности, когда нижележащий бетон еще мягкий, может затруднить отделку и привести к образованию волнистых или потрескавшихся поверхностей.

    Когда замедлитель схватывания может не потребоваться

    • Короткие перевозки с быстрым размещением
    • Холодное влажное внутреннее расположение
    • Жилой или коммерческий подвал
    • Высокая скорость заливки

    Текучесть

    Суперпластификаторы

    (высокодисперсные водоредукторы) могут превратить бетон с низкой или нормальной осадкой в ​​текучий бетон с высокой осадкой, который можно укладывать с минимальной вибрацией или без нее.Однако изменение спада обычно длится всего от 30 до 60 минут в зависимости от марки и дозировки.

    Регуляторы воды высокого диапазона попадают в классификацию ASTM C494 Тип F или Тип G. В любом случае их можно использовать для получения очень высоких оседаний без сегрегации, что является идеальной ситуацией, когда требуется повышенная текучесть из-за перегруженности арматуры.

    Другие области применения и преимущества высокопроизводительных редукторов воды включают:

    • сложное размещение стен
    • узкие формы
    • секции с блокировками, проходами или закладными деталями
    • перекачивание на большие расстояния по вертикали
    • быстрое бетонирование
    • Увеличенная высота подъема и дистанция свободного падения

    Примечание: Увеличенная толщина бетонной смеси означает, что формы должны быть плотными, чтобы предотвратить утечку даже через небольшие стыки, которые могут привести к ребрам и обесцвечиванию.

    Суперпластификаторы типа F

    Добавляется на стройплощадке и сохраняет текучесть бетона в течение короткого периода времени. В какой-то момент бетон быстро потеряет осадку.

    Суперпластификаторы типа G

    Может быть добавлен либо во время дозирования, либо на стройплощадке. Эта добавка замедлит схватывание, но заставит бетон оставаться текучим в течение более длительного периода времени, что может задержать отделку. Если время перевозки особенно велико, на заводе можно добавить тип G.Однако, если доставка задерживается слишком долго, последствия могут быть уменьшены. Для восстановления пластичности смеси возможно повторное дозирование, и следует строго соблюдать рекомендации производителя.

    Корпорация Fritz-Pak в Далласе, Техас

    Улучшители чистовой обработки

    Водоредуцирующие добавки среднего класса могут использоваться в качестве улучшителей отделки бетона, особенно в коммерческих и жилых зданиях, а также в формованном бетоне.

    MRWR производят менее проницаемый, более прочный бетон и бетон со значительно ранним и предельным пределом прочности на сжатие.

    Примечание по бетонированию в холодную погоду

    Падение на 20 o F может удвоить время, необходимое бетону для схватывания. Ускорители ASTM C494 типа C или комбинация ускорителей типа F и восстановителей воды могут быть решением.

    Записка по бетонированию в жаркую погоду

    Как правило, каждое повышение температуры окружающей среды на 10 o F снижает оседание примерно на 1 дюйм. Повышение температуры смеси на 30 o F может вдвое сократить время схватывания, увеличить потребность в воде и уменьшить 28-дневное сжатие. прочность целых 25%.

    Переход с ASTM C494 типа A на водоудерживающую и замедляющую схватывание смеси типа D может быть частью эффективного плана бетонирования в жаркую погоду.

    Производство легко протекающей и самовыравнивающейся жидкости для засыпки

    Контролируемый материал низкой прочности (CLSM)

    Этот материал обеспечивает жидкую засыпку, которая легко течет и самовыравнивается.

    Хотя он стоит больше, чем сухой гранулированный материал, его не нужно помещать в подъемники, разложить и уплотнять после каждого подъема.

    Это жидкая смесь, состоящая из портландцемента, воды, мелкозернистого заполнителя и / или летучей золы, и может включать в себя добавку для улучшения текучести, снижения плотности, устранения сегрегации и оседания, а также контроля развития прочности в приложениях , где требуются будущие земляные работы.

    Типичная 28-дневная прочность на сжатие составляет от 50 до 200 фунтов на квадратный дюйм, а плотность составляет от 115 до 145 фунтов на квадратный фут.

    Найдите поставщика или производителей

    История бетона | BigRentz

    Бетон настолько дан в нашей повседневной жизни, что вошел в наш словарный запас: когда мы говорим что-то «конкретное», мы имеем в виду, что это существенное, прочное, постоянное, на что можно рассчитывать.Кроме того, большинство из нас проводит свою жизнь на бетоне и вокруг него, на тротуарах и дорогах, внутри зданий и сооружений, построенных из чудесного материала. Без бетона развитый мир выглядел бы совершенно иначе.

    Вы когда-нибудь задумывались, откуда произошел бетон и как он стал повсюду в современной жизни? За этим важным строительным материалом стоит долгая и впечатляющая история, начавшаяся тысячи лет назад, еще до египетских пирамид, охватывающая время беспрецедентных построек римлян и переходящая в современное строительство.

    Мы рассмотрим, что такое бетон (а что нет), как он появился, как он сыграл роль в создании великих городов и монументальных зданий мира и как он влияет на нашу повседневную жизнь.

    Важное различие: цемент и бетон

    Прежде чем углубляться в историю бетона, следует прояснить одно важное заблуждение: бетон — это не то же самое, что цемент. Хотя эти два слова часто путают друг с другом, есть одно главное различие: цемент — это компонент бетона.

    Цемент изготавливается из различных комбинаций известняка, глины, ракушек, мела, сланца, сланца, кварцевого песка и иногда даже доменного шлака или железной руды. Эти ингредиенты измельчаются, затем нагреваются при высоких температурах, в результате получается материал под названием клинкер . В клинкер добавляется гипс, затем вся смесь тонко измельчается до цементного порошка.

    Просто добавьте воды, и процесс станет интересным. Гидратация — это процесс, который происходит, когда содержащиеся в цементном порошке минералы — кальций, кремний, алюминий, железо и другие — образуют химические связи с молекулами воды.По окончании этого процесса вода испаряется, а паста высыхает, оставляя после себя эти связи, организованные в виде камня.

    Итак, бетон представляет собой смесь этого цементно-водного теста и песчано-каменного заполнителя. Паста покрывает поверхность песка и камней, связывая их вместе в смесь, известную как бетон. В жидкой жидкой форме бетону можно придать практически любую форму, которую пожелает строитель — лист, колонна, блок, плита, арка, чаша и т. Д. Когда вода в пасте высыхает, бетон становится твердым, как скала, и удерживает эта форма.

    Цемент обычно составляет около 10-15 процентов бетонной смеси. Почти все типы бетона используют портландцемент. Это не торговая марка, а признанный вид цемента, который широко используется в отрасли (например, «нержавеющая сталь» или «стерлинговое серебро»). Его создатель назвал свою смесь в честь высококачественных строительных камней, найденных в соседнем карьере в Портленде, Англия.

    Неопровержимые факты о бетоне

    Бетон стал настолько популярным (и остался таким) благодаря своим трем выдающимся качествам: пластичности, прочности и экономичности.В мокром состоянии бетон может принимать практически любую форму, вписываться в любое пространство, заполнять практически любые пустоты, покрывать практически любую поверхность. Но как только он высыхает и застывает, он сохраняет свою форму, становясь со временем сильнее, тверже и устойчивее.

    Бетон, изготовленный с правильной концентрацией и в правильных условиях, может быть водонепроницаемым, штормостойким и огнестойким. И благодаря этой прочности он длится практически вечно. Через миллион лет, когда вся сталь, из которой мы строили наш мир, проржавела, а дерево превратилось в пыль, останется бетон.

    Но бетон не только прочен; он также достаточно экономичен, чтобы поддерживать мировую промышленность, производящую более 2 миллиардов тонн бетона в год, что в среднем составляет около 5 тонн на человека в год, и без того шокирующий темп, который, как ожидается, удвоится к 2050 году! Только Китай залил больше бетона для строительства в период с 2011 по 2014 год, чем США за последние сто лет.

    Если вы находите эти факты удивительными, следуйте за нами по «бетонной дороге» через хронологию других увлекательных достижений.Вы увидите, как бетон стал материалом, который буквально проложил путь к жизни, какой мы ее знаем сегодня.

    Бетонная история сквозь века

    Итак, как мы пришли к текущему состоянию бетона? В процессе эволюции, как и многие другие средства строительства и развития. Во-первых, древние люди сделали открытия о природных материалах, которые они могли использовать для улучшения основных частей своей инфраструктуры — домов, заборов, колодцев и т. Д. Поколения, которые следовали за ними, основывались на этих знаниях, внося улучшения то здесь, то там, пока не наступила индустриальная эра. и ускорили застройку до нынешнего уровня.

    Происхождение и предшественники

    12 миллионов лет назад — Природный цемент

    На земле, которая сейчас является Израилем, самовозгорание вызвало реакции между известняком и горючими сланцами, в результате чего образовались естественные отложения «природного цемента», которые сделают возможным образование бетона в будущем.

    10 000 до н.э. — Ранняя известняковая структура

    Известняк, также часто называемый «известью», играет самую раннюю роль в истории бетона в качестве основного ингредиента цемента и использовался на протяжении тысячелетий.Гебекли-тепе на территории современной Турции, предшествующий еще одному массивному каменному храму, Стоунхенджу, на 6000 лет был самым ранним известным строением из известняка. Известняк составлял Т-образные столбы этого храма, которые были построены и вырезаны доисторическими людьми, которые еще не разработали металлические инструменты или даже керамику.

    6500 до н.э. — Пустыня цистерны

    Первые бетонные сооружения, секретные подземные цистерны для хранения дефицитной воды, были построены набатейскими или бедуинскими торговцами, которые создали небольшую империю в пустынных оазисах южной Сирии и северной Иордании.Некоторые из этих цистерн все еще существуют в тех областях сегодня.

    5600 до н.э. — Сборное железобетонное перекрытия

    На территории бывшей Югославии, в районе Лепенски Вир на берегу Дуная, в середине 1960-х годов были найдены хижины с подобием бетонных полов. Известковый цемент, который использовался, вероятно, прибыл из месторождения вверх по реке и был смешан с песком, гравием и водой, чтобы напоминать бетонные смеси нашего времени.

    Памятники старины

    3000 до н.э. — Египетские пирамиды

    Известняковые камни или бетонные блоки? Несмотря на некоторые горячо обсуждаемые предположения о том, что блоки в египетских пирамидах были сформированы из более раннего типа бетона более 5000 лет назад, в области археологии более широко распространено мнение, что блоки известняка были доставлены из близлежащих карьеров.Чтобы сделать раствор для скрепления блоков, строители смешали солому с грязью, содержащей измельченный известняк, гипс и глину.

    1400-1700 до н.э. — минойские постройки на Крите

    Минойское общество на острове Крит, предшественники греков и считающееся первой европейской цивилизацией, использовало строительный материал, состоящий из смеси глины и вулканического пепла, называемого пуццолана , для строительства полов, фундаментов и канализации.

    * 1300 до н.э. — Первое «известковое» покрытие

    Ближневосточные строители обожгли известняк и смешали его с водой, а затем использовали эту смесь для покрытия наружных поверхностей своих стен из толченой глины.Когда смесь вступала в реакцию с воздухом, она образовывала твердую защитную поверхность — и закладывала основу, так сказать, для современных разновидностей цемента.

    1000 до н.э. — греческие гробницы

    Микенцы использовали свою раннюю форму цемента для строительства гробниц. Некоторые из них вы можете увидеть сегодня на Пелопоннесе в Греции.

    770-476 до н.э. — Великая китайская стена

    Северные китайцы использовали форму цемента для постройки лодок и их участка Великой стены.На протяжении веков строительства стены использовались материалы, используемые для ее строительства, включая тростник, ивовые ветви, дерево, уплотненный песок, грязь и 100 миллионов тонн камня и кирпича. Там, где они не были зацементированы известняковым раствором, они скреплялись раствором из клейкого липкого риса.

    * 700 до н.э. — Печи, строительный раствор и гидравлическая известь

    Те же бедуины, которые первыми создали подземные цистерны, позже построили печи для производства рудиментарного вида гидравлической извести — цемента, который затвердевает под водой — для водонепроницаемого раствора, который продвинул строительство домов, полов и новых водонепроницаемых цистерн под землей.

    От Римской Империи до Возрождения

    300-500 гг. Н.э. — римская архитектура

    Римляне начали с того же сырья, что и минойцы — вулканического пепла, найденного недалеко от Помпеи и горы Везувий, который они использовали для уплотнения смеси обожженного известняка, измельченных камней, песка и воды, что позволило им построить пандусы и террасы. , и дороги, которые в конечном итоге соединили всю империю. Выливание смеси в формы вскоре позволило строителям создавать своды и купола, а также арки культовых акведуков и бань империи.Римский бетон пережил землетрясения, удары молний, ​​удары морских волн и тысячи лет выветривания.

    82 нашей эры — Колизей

    После гражданской войны в Риме император, известный как Веспасиан, намеревался построить самый большой театр в мире на более чем 50 000 мест. Сегодня мы знаем первый в мире стадион, построенный 1937 лет назад, как «Колизей». Около трети сооружения все еще стоит почти два тысячелетия спустя и является культовым символом Римской империи.

    117-125 нашей эры — Пантеон — и потеря бетона

    Римский Пантеон, который скоро отметит свое 1900-летие, как никогда прочен. Неармированный бетонный купол храма был вдвое шире и выше любого купола, когда-либо созданного в то время, и его длина составляла 143 фута со знаменитым окулусом в центре. Его гигантский вес поддерживается невероятно толстыми бетонными стенами и восьмью цилиндрическими сводами, усиленными кирпичом, но без внутренней опоры.

    Современные инженеры не осмелились бы построить неармированный купол такого размера, и они, возможно, никогда не узнают секрет долговечной стабильности Пантеона.Мы действительно знаем, что инженеры императора Адриана скорректировали рецепты бетона, используя больше вулканического пепла, чем камня, чтобы сделать купол легче, и больше каменного заполнителя в стенах для более тяжелой арматуры. Но когда Римская империя пала в 476 году нашей эры, беспрецедентный римский рецепт приготовления бетона был утерян для мира.

    1507 — Возрождение — Мост Нотр-Дам

    Сразу после Средневековья итальянский монах по имени Джованни Джокондо построил мост Пон-Нотр-Дам в Париже, используя информацию, оставшуюся от древнеримского рецепта цемента.Примерно через 250 лет это сооружение было снесено, потому что дома, построенные на мосту, добавили слишком большого веса. Джокондо вошел в историю как единственный человек, пытавшийся строить из бетона в эпоху Возрождения.

    Достижения в бетоне

    Усовершенствования XVI века

    Каменщик в Андернахе, Германия, попытался смешать вулканический пепел под названием trass с известковым раствором. Полученный материал был водостойким и прочным, и цепная реакция, начатая с открытием, привела к созданию современного цемента.

    Торговля бетоном XVII века

    В 17 веке голландцы (которые уже были мастерами строительства в воде) продали трасса Франции и Великобритании для использования в зданиях, которым требовались водонепроницаемые свойства. Две соперничающие страны немедленно начали конкурировать за создание собственных гидравлических строительных материалов.

    * 1793 — Современное производство извести для цемента

    Когда британскому инженеру-строителю Джону Смитону было поручено построить новый маяк на скалах Эддистоун в Корнуолле, Англия, он приступил к поискам самого прочного и водонепроницаемого строительного материала, который он мог найти.Обнаружив поблизости известняк с высокой концентрацией глины, он обжигал его в печи и превратил в клинкер. Он измельчил его в порошок и смешал с водой, чтобы получить пасту, из которой он построил маяк.

    В процессе — и спустя более чем 1000 лет после того, как секреты бетона были потеряны — Смитон заново открыл, как производить цемент. Вскоре производители начали продавать его открытие как «римский цемент». А маяк Эддистон простоял почти 130 лет, пережив скалы, которые выветрились из-под него.

    * 1824 — Изобретение портландцемента

    Англичанин Джозеф Аспдин усовершенствовал процесс, тщательно смешав известняковый мел с глиной и обжигая смесь в печи до удаления углекислого газа. Он также нагревает глинозем и кремнезем до тех пор, пока материалы не станут стеклоподобными, затем измельчил их и добавил в известняковую смесь вместе с гипсом.

    Полученная химическая комбинация кальция, кремния, алюминия, железа, гипса и других минеральных ингредиентов составляет отличную формулу портландцемента, основного ингредиента бетона.Аспдин назвал результат «портландцемент», потому что он напоминал высококачественные строительные камни, добытые в соседнем Портленде, Англия.

    * 1836 — Испытания на прочность

    Первые испытания бетона на растяжение и сжатие прошли в Германии. Прочность на разрыв — это способность противостоять растяжению или растяжению; Прочность на сжатие — это способность противостоять сжатию или сдвигу.

    * 1850-е — Армирование стальной сеткой запатентовано

    Французский садовник Жозеф Монье успешно экспериментировал с заливкой бетона на стальную сетку.(Бетон и сталь расширяются с одинаковой скоростью при нагревании, что делает их идеальным сочетанием). Монье запатентовал несколько вариантов своего изобретения для использования с вагонами-шпалами, строительными плитами и трубами. Железобетон намного прочнее и практичнее неармированного материала. Он может перекрывать большие промежутки, позволяя бетону взлетать в виде мостов и небоскребов.

    * 1880-е годы — Армирование железными прутьями

    Калифорнийский инженер Эрнест Рэнсом начал испытания бетона и 2-дюймовых железных прутьев, чтобы увидеть, будут ли материалы сцепляться.Когда они это сделали, Рэнсом пошел еще дальше, скрутив железные прутья, чтобы создать арматуру, вокруг которой он мог «построить» бетон любой желаемой формы — эксперимент, который также сработал. Сегодня мы называем эту систему арматурным стержнем или арматурой, хотя современные инженеры обычно используют сталь вместо железа.

    Система Рэнсома скоро будет использоваться в коммерческих зданиях, на дорогах, мостах и ​​даже в первых небоскребах. Знаменитый архитектор Фрэнк Ллойд Райт начал применять технологию арматурного бетона в современной архитектуре.Некоторые из самых известных зданий Райта, в том числе Храм Единства в Оук-Парке, штат Иллинойс, который считается первым современным зданием в мире; и Fallingwater в Милл-Ран, штат Пенсильвания, его самые знаменитые работы, были сделаны из железобетона.

    * 1880-е годы — Запатентованная сталь для предварительного напряжения

    Процесс предварительного напряжения стали был запатентован, чтобы сделать бетон более прочным и позволить инженерам использовать меньше стали и бетона.

    Современные бетонные конструкции

    С тех пор, как Рэнсом разработал использование арматуры, из бетона были построены все типы монументальных зданий и объектов инфраструктуры.Панамский канал, бункеры времен Второй мировой войны и знаменитый Сиднейский оперный театр делят строительный материал с некоторыми из самых сложных и самых дальновидных зданий в мире.

    1889 — Первый железобетонный мост — мост через озеро Алворд, Сан-Франциско

    Мост через озеро Алворд был построен в 1889 году в Сан-Франциско, Калифорния. Первый железобетонный мост, он пережил землетрясение в Сан-Франциско 1906 года и другие без повреждений. Он существует до сих пор, спустя более 100 лет после постройки.

    1891 — Первая бетонная улица в Америке — Беллефонтен, Огайо

    В 1891 году человек по имени Джордж Бартоломью построил первую бетонную улицу в Америке в Беллефонтене, штат Огайо. Сегодня проницаемый бетон пропагандируется как лучшее и самое экологически чистое покрытие для улиц.

    1903 — Первое бетонное высотное здание — The Ingalls Building, Цинциннати

    В Цинциннати в 1903 году система Рэнсома позволила построить первое бетонное высотное здание — 16-этажное здание Ingalls Building.Эта невероятная высота сделала небоскреб одним из величайших инженерных достижений своего времени.

    1899 — Мост через реку Вьен

    Мост через реку Вьен в Шательро, Франция, построенный в 1899 году, является одним из самых известных железобетонных мостов в мире.

    1908 — Бетонные дома — Юнион, Нью-Джерси — спроектирован и построен Томасом Эдисоном

    Первые бетонные дома в стране были спроектированы и построены в Юнион, штат Нью-Джерси, никем иным, как Томасом Эдисоном.Эти дома существуют и сегодня.

    * 1913 — Поставка первой готовой смеси — Балтимор

    Первая партия «готовой смеси» доставлена ​​в Балтимор. Смешивание бетона в одном месте (центральный завод), а затем его доставка грузовиком для использования на стройплощадке, стало революцией в бетонной промышленности.

    * 1915 — Цветной бетон — L.M. Scofield, первая компания по производству цветного бетона

    Линн Мейсон Скофилд основала L.M. Scofield, первую компанию по производству красок для бетона.Их продукция включала отвердители цвета, цветной воск, интегральный краситель, герметики и химические пятна.

    * 1930 — Воздухововлекающие агенты — устойчивость к повреждениям от замерзания и оттаивания

    В 1930 году воздухововлекающие агенты были впервые использованы в бетоне, чтобы противостоять повреждениям от замерзания и оттаивания — явное благо для методов строительства в холодную погоду в Соединенных Штатах и ​​во всем мире.

    1936 — Плотина Гувера — крупнейший бетонный проект, когда-либо завершенный на то время

    Плотина Гувера расположена на границе Аризоны и Невады.Построенная в 1936 году для сдерживания могущественной реки Колорадо, плотина состоит из 3,25 миллиона кубических ярдов бетона, а еще 1,11 миллиона было использовано для строительства электростанции и окружающих сооружений.

    1956-1992 — Американская система автомагистралей между штатами

    Все дороги Америки в системе автомагистралей между штатами сделаны из железобетона.

    1963 — Актовый зал Университета Иллинойса — первый бетонный спортивный купол

    Первая спортивная арена с бетонным куполом была построена на территории кампуса Иллинойского университета в Урбана-Шампейн в 1963 году.Арена, известная как Актовый зал, выглядит как летающая тарелка и вмещает более 16 000 человек в идеальном бетонном круге.

    * 1970-е — Армирование волокном — способ усиления бетона

    Армирование волокном, в котором стекло, углерод, сталь, нейлон или другие синтетические волокна смешиваются с влажным бетоном перед заливкой, было введено как способ укрепления бетона. Волоконное армирование может использоваться для усиления зданий, а также наружных элементов, от проездов, плит и тротуаров до бассейнов, террас и террас.

    1992 — Самое высокое железобетонное здание — Чикаго

    65-этажный небоскреб по адресу 311 South Wacker Drive в Чикаго был самым высоким в мире железобетонным зданием на момент его постройки. Постмодернистская структура известна только по адресу.

    Будущее бетона?

    * Современная эпоха — при снижении производительности получается бетон более низкого качества

    Когда-то бетон считался ответом на мировые строительные проблемы; он податлив во влажном состоянии, прочен и долговечен в сухом виде и достаточно дешев, чтобы сделать практически все, что вы захотите.

    Проблема в том, что это не навсегда. По крайней мере, он не остается неповрежденным и не жизнеспособным постоянно (хотя и не легко ломается). Несмотря на всю свою впечатляющую прочность на разрыв, современный бетон может сохранять целостность без капитального ремонта или замены в лучшем случае около века. Сегодняшний железобетон не может сравниться с «римским бетоном».

    Особенно, если железобетон изготавливается дешево — скажем, с несбалансированной смесью, некачественными ингредиентами или небрежной заливкой — он может начать распадаться изнутри.По мере выветривания вода постепенно просачивается сквозь крошечные трещинки и направляется к стали в середине. По мере того как бетон, окружающий его, застывает, арматурный стержень окисляется и может расшириться настолько, что приведет к растрескиванию бетона, который он должен поддерживать.

    Соленая вода особенно вредна для арматуры, поскольку соль разъедает сталь в течение пяти десятилетий. Повторяющиеся циклы замерзания и оттаивания также могут создавать и расширять трещины, особенно на бетонных дорогах. Распространение соли действительно препятствует образованию льда, но она действует в тандеме с влагой, нанося такой же вред арматурному стержню, как если бы морская вода постоянно омывала его.

    * Будущее — Возможные улучшения для обслуживания и производства бетона

    Существует множество новых методов улучшения бетона, включая специальные методы обработки, предотвращающие проникновение воды в сталь. Другие достижения являются ответом на растущее внимание во всем мире к устойчивости: «Самовосстанавливающийся» бетон содержит бактерии, которые выделяют известняк, закрывая любые возникающие трещины. Смесь для «самоочищающегося» бетона наполнена диоксидом титана, который разрушает смог и сохраняет белизну бетона.Усовершенствованные версии этой технологии могут даже дать нам уличные поверхности, которые очищают выхлопные газы от автомобилей.

    Кроме того, в недавнем отчете говорится, что мы можем повторить рецепт римского бетона (который, несмотря на более низкую прочность на разрыв, демонстрирует беспрецедентную долговечность). Римский бетон не только водонепроницаем; Было обнаружено, что он становится сильнее при контакте с морской водой. Ученые предполагают, что микроскопические кристаллы растут в древнем бетоне, когда он погружен в воду, что делает его еще менее уязвимым для выветривания.

    Хотя они до сих пор не смогли полностью собрать утраченный рецепт, исследователи знают, что вулканический пепел пуццолана имел фундаментальное значение для прочности древнеримского бетона. Недавно объявленный проект будет экспериментировать с подобным вулканическим пеплом у побережья Калифорнии, чтобы попытаться реконструировать процесс, который позволил создать самый прочный бетон в истории.

    Если это произойдет, сочетание секретного рецепта Рима по изготовлению бетона и современных методов проектирования арматуры может снова произвести революцию в использовании бетона, а также в мировой инфраструктуре и архитектуре.

    Источники:

    https://www.nachi.org/history-of-concrete.htm

    https://www.everreadymix.co.uk/news/a-history-of-concrete-infographic-by-ever-readymix/

    https://www.concretenetwork.com/concrete-history/

    https://www.chinahighlights.com/greatwall/fact/how-the-great-wall-was-built.htm

    https://www.citylab.com/design/2017/08/undercover-economist-cement-shaped-the-modern-economy/537780/

    https: //www.citylab.ru / design / 2014/11 / вступление в эпоху разрушения бетона / 382888/

    https://www.concretenetwork.com/concrete/whatis/

    https://www.cement.org/cement-concrete-applications/how-cement-is-made

    https://www.popularmechanics.com/technology/infrastructure/a28502/rock-solid-history-of-concrete/

    https://www.smithsonianmag.com/history/gobekli-tepe-the-worlds-first-temple-83613665/

    https: // www.worldscientific.com/doi/pdf/10.1142/9789813145740_0001

    https://www.revolvy.com/page/Ernest-L.-Ransome

    https://explorecu.org/items/show/268

    Похожие сообщения

    Прочность бетона на сжатие сердечника: 6 факторов

    Факторы, которые влияют на прочность бетона на сжатие сердцевины, следующие: 1. Влага и пустоты 2. Отношение длины к диаметру сердцевины 3.Диаметр сердечника 4. Положение вырезанного бетона в конструкции 5. Направление сверления 6. Влияние возраста.

    Фактор № 1. Влага и пустоты:

    Состояние влажности сердечника влияет на измеренную прочность. Было замечено, что насыщенный образец имеет значение на 10-15% ниже, чем сопоставимый сухой образец. Таким образом, при оценке фактической прочности монолитного бетона необходимо учитывать условия относительной влажности сердечника и монолитного бетона.

    Пустоты в основном бетоне уменьшают измеренную прочность. Петерсон обнаружил, что отношение прочности сердечника к прочности стандартного цилиндра в одном и том же возрасте всегда меньше 1,0 и уменьшается с увеличением прочности цилиндра. До прочности цилиндра 20 МПа это чуть меньше 1 и 0,7 при прочности 60 МПа.

    Фактор № 2. Отношение длины / диаметра сердечника:

    Было замечено, что по мере увеличения отношения l / d измеренная прочность уменьшается из-за влияния формы образца и распределения напряжений во время испытания.Для установления связи между прочностью сердечника и стандартной прочностью куба за основу расчета берется отношение l / d = 2,0.

    Фактор № 3. Диаметр сердечника:

    Диаметр сердечника может влиять на измеренную прочность и изменчивость. Измеренная прочность бетона уменьшается с увеличением размера образца. Этот эффект значительный. Однако этот эффект будет небольшим для размеров более 100 мм, но для меньших размеров этот эффект будет значительным.

    Фактор № 4. Положение вырезанного бетона в конструкции:

    Сердечники, взятые около верхней поверхности, обычно имеют самую низкую прочность, будь то колонна, балка, стена или плита. С увеличением глубины ниже верхней поверхности прочность увеличивается, но на глубине более 300 мм дальнейшего увеличения прочности не происходит. Разница может составлять от 10 до 20%. В случае плит эта разница увеличивается из-за плохого отверждения.

    Фактор № 5. Направление бурения:

    Из-за эффекта наслоения измеренная прочность образца, просверленного вертикально относительно направления заливки, вероятно, будет больше, чем прочность образца, просверленного горизонтально из того же бетона. В литературе сообщается о среднем значении 8% этой разницы.

    Фактор № 6. Влияние возраста:

    Было замечено, что бетон на месте не набирает прочность через 28 дней.Испытания высокопрочного бетона показали, что, хотя прочность сердечника увеличивается с возрастом до 1 года, но остается ниже, чем стандартная прочность цилиндра в течение 28 дней, как показано на рис. 19.6 и таблице 19.3. С другой стороны, Petersons предполагает увеличение прочности сердечника по сравнению с 28-дневной прочностью цилиндра на 10% через 3 месяца и 15% через 6 месяцев. Таким образом, нелегко обсудить влияние возраста, но при отсутствии определенного влажного отверждения не следует ожидать увеличения прочности.

    Кривая А на рис.19.6 показано развитие прочности стандартного цилиндра. Кривая B показывает прочность хорошо затвердевшего сердечника плиты, испытанного в сухом виде. Кривая C показывает изменение прочности хорошо затвердевшего сердечника плиты, испытанного во влажном состоянии.